Sunteți pe pagina 1din 63

Downloaded From OutlineDepot.

com

EVIDENCE OUTLINE SUMMER 2008- CRIBARI


Possible Exam Questions: 3 HOUR EXAM. 9am!
If you were the lawyers for Her what would you do after the Giles Decision?
MN Sup Ct didnt find intent and just said that regardless of whether the statements are
testimonial that he forfeited his rights by murdering her. This is WRONG!!!
No forfeiture after Giles, so go back to trial court to try to get intent and/or go back to try to get
the hearsay is testimonial.
But all 7 justices agreed on the result, they just disagreed on how to get there.
Does Giles provide for a domestic violence exception to the confrontation clause?
Breyer and Stevens are potentially pushing for a purpose requirement, more stringent than
intention.
If you can infer an intent from the murder to keep the witness from testifying
Gingsberg and Souter were sounding that you shouldnt start inferring the intent aspect. Even if
no witness there may be a case coming, domestic violence case you would not have this.
4 of the majority are holding that the 6th Amend. Means what it meant in 1791 at the founding.
Exam Overview- Hints/TipsSuggestions:
He will check email Thursday and Friday Morning, after that he will not be available
Short answer, multiple choise, true/false, 150 word answers, short answer and tell why (briefly explain, 25-50 words),
The ultimate question is relevance
If hearsay and inadmissible it is not relevant to this trial because it is dangerous as(unfair, ects)
Rules of General Relevance:
401- evidence that has ANY tendency to prove or disprove a fact of consequence
It has something to offer to help you find or not find a fact provided you are allowed to prove that
fact
402 if evidence is relevant, then it is admissible (unless some other rule says it isnt admissible)
403- if the evidence creates the danger that whatever it might prove might do it in an unfairly prejudicial way it is kept
out- inadmissible (danger of misleading, confusing, ect must substantially outweigh the probative value of the fact to be proven by
the evidence)
Rules of Limited Relevance (for a particular thing): Almost always a limiting instruction will be given! ( 105):
404-405- Character- 404a- cant offer proof of character to prove that D acted in conformity with that character, but you
are allowed to do it in three instances: 1. When D puts her character in issue, 2. When D puts the victims character in issue, 3.
Whenever the witness takes the stand- but the only character that is admissible is character for truthfulness. See 607-609.
405- tells you how to do 404, just like 608.
For the exceptions you can only prove character with opinion and reputation evidence
Mickleson case
When character is an element of the crime then you can prove character with specific instances of
conduct
406
407-410- offers of compromise, subsequent remedial measures, plea agreements, ect. (encourage these and give them
protection to avoid court)
Hearsay- Article 8
801a-c- out of court statement recounted in court to prove the matter of the fact asserted- general definition
801d- limited definition, 801d1a- inconsistent statements, 801d2- prior consistent statements and motive to prior statement
(Thome case)if they look sound and feel like hearsay
802- if it is not hearsay it is admissible, and if hearsay is not admissible unless 803, 804 (1-3, 6)(available and unavailable
declarants
803(1-6, 8, 22-23)- present sense, excited utterances, prior convictions, business records, ect,even if it is false, as long as it
meets the requirements it comes in
805-805- multiple hearsay, you must address each chain, you can impeach the declarant
807- not an answer on this exam!!!
Article 2, 3, and 7- Not on this exam
Article 9 and 10- Authentication and Best Evidence
1
Downloaded From OutlineDepot.com

You must demonstrate that it is relevant- sufficient to establish that the evidence is what it says it is.
Writing, recordings, data compilations, ect (what ever article 10 covers)- you must offer an original or a duplicate of the
original and not a copy.
901- examples (argue 104- weight and credibility), photos- Are you introducing the most reliable evidence possible
Article 11- when it doesnt apply
Article 5- Privilege
Psychotherapist-patient (federal law)- Menendez Broths. Cases
Lawyer/ Client (Zolan-crime fraud exception)
Marital Privilege- spouse cant be kept off the stand but may be able to keep her quiet regarding some issue (belongs
to the witness spouse.)
Doctor/Patient
Newspaper Writers sources- no constitutional that a reported enjoys, but reporter shield laws tries to create these
state by state.
All are strictly construed
Also depends on Whether the case is criminal vs. civil, Ferber case (is it necessary, I s there an
alternative?
Article 6- ONLY applies for witnesses:
Refresh recollections, incompetency rules, no presumption of requirements to be a witness.
If no personal knolwed inadmissible, 615- witnesses excluding witnessed from staying in the court
612 similar to 803 F
613 prior inconsistent statements
607- overrules witness voucher rule
608a- witness that testifies automatically puts in his or her character for truthful in issue
608b- limited ability to ask questions re specific instantences of conduct, but no extrinsic evidence to prove the
matter stated re the witness
609a1- if witness is not D you can impeach with a felony to prove that the witness is a liar.
609a2- can offer prior conviction, you can impeach to the jury with that the previous conviction. Still must locate
exception- no discretion on behalf of the judge shall!!!
Ten year SOL on convictions
Article 1
103- object or waiver
104
105-
Crawford- criminal trial 6th amend right to confront witnesses
Up until 2004 it was the U.S. v. Roberts- super reliabilily- unavailability was required and a guarantee that the hearsay
exception was well founded.- unavailability as a requirements was done away with.
Ohio v. Roberts got it right for the wrong reasons
Whether it was admissible because it was cross-examined
Crawford says we only look at the hearsay that the 6th amendment is aimed at testimonial hearsay- and whether that was
cross-examined. Look to pg. 330
If testimonial hearsay then the test is the following:
Declarant is unavailable and
Declarant was previously cross-examined in court.
Davis and Hammon- looked at what might not be testimonial hearsay. A distinction was made between an ongoing
emergency and investigation and or interrogation for the purposes of prosecution
803(6 and 8)- Lab reports and the 6th amendment- are those reports testimonial hearsay? Court began to distinguish between
facts and conclusions (testimonial) and some courts took the opposite view (melendas v. dias
Giles- Forfeiture Doctrine down away with- 804b6 in the rules, but as a constitutional matter this is the case, if the government can
show that the D made the declarant unavailable so that the declarant couldnt testify then the statements come in. Sup. Ct thought that
you shouldnt benefit from killing a witness, not just a person
Requires showing of intent to prevent a witness from testifying.
Cant do what MN rule said which was a murder (a person) to the 6th amendment
2
Downloaded From OutlineDepot.com

I.What is a trial? A dispute resolution process


A. Criminal Trial- trying to determine guilt or innocence in order to determine whether the state has the right to punish
the D.
B. An Adversarial Process-
C. A Competition Over Justice- The rules of evidence are there to ensure fair competition.
1. A competition of words
D. Do Not think outside the box in Evidence
1. The more you can constrict the box the more control you will have
2. Learn the rules, apply the rules
3. If it is not in the record, it doesnt exist!!!
II. The Record- What it Means and How It Is Made
A. Meaning and Purpose of the Trial Record- A reviewing court can only act on the formal record of the trial that
has been officially transmitted to it by the clerk of the trial court
1. Made up of suit papers- pleadings, motions, briefs, orders, etc.
2. Made up of 3 parts
a. Litigations paperwork
b. Verbatim transcript of hearings, conferences, and trial testimony
c. Tangible exhibits that the parties offered into evidence
B. Conduct of Lawyers that Hampers Court Reporters
1. Echoing - constantly repeating the witness
2. Overlapping - more than one person talking at the same time
3. Numbers - specify what numbers refer to when speaking
4. Proper Names - enunciate proper names slowly and clearly; spelling
5. Exhibits - refer to exhibits by the entire name/title
6. Indications and Gestures - should clarify orally
7. Off the Record - clear indication when record resumes
8. Sidebar Conferences - should make sure reporter can record/hear
9. Abtuse Terminology - specialized cases should supply a glossary
10. Reading Testimony into the Record - specify questions/answers
C. Requesting the Making of Record
1. Many aspects of litigation will not be part of the record unless counsel supplies a court reporter - litigants
have obligation to provide and pay for services
2. Make the record so that an appellate court can review the courts decision, so counsel will attempt to get
what the purported response would have been if the evidence would have been allowed.
D. Statements for the Record
1. When either inaudible or nonverbal responses are given, counsel may state for the record what the witness
meant in their response
E. Stipulations- a term of art
1. Generally written out and signed by both parties
2. Def - voluntary agreement entered into between counsel for the parties to a litigation respecting some
matter that is before the trial court. Taking an issue out of the case.
3. Can relate to either procedure or evidence
a. Evidentiary - acts to admit or concede specific facts, relieving the party of the burden of making
full-scale proof
b. Procedural - example: stipulations for the discovery process must be in writing (Fed.R.Civ.Pro.
29)
4. Counsel is ordinarily required to accept any stipulation offered by the opposing side which unequivocally
concedes everything that counsel would be entitled to show by making full proof
II. Offering Evidence
A. Direct Examination of Witnesses

3
Downloaded From OutlineDepot.com

1. Direct questions on direct examination- No leading questions


2. Leading Questions - not usually allowed except... A question that suggests the answer.
a. Preliminary matters that do not go to the heart of the case and provide a transition from one
subject to another
b. Permitted with respect to undisputed matters where the question is used as a connective
c. An adverse or hostile witness can be asked leading questions
d. Allowed during direct examination when a witness gives surprise answers
e. Allowed in connection with a witness of limited understanding
f. Witness whose recollection has been exhausted but who possesses additional information of a
relevant sort
g. Hypothetical questions to an expert witness where they provide a factual basis for the experts
opinion
3. Compound and Otherwise Confusing Questions - should be avoided
4. Narrative Answers are not allowed- Or questions that call for narrative answers are objectionable.
5. Rule 103- if an objection is not timely made it is waived. Object now or waive. Plain error is the
standard used by the appellate court when there is continued questions allowed and then all the sudden
there is a question.
6. Evidentiary Error is reviewed on a Abuse of Discretion Error- Appellate court must accept the trial
courts decision unless no court could reasonably come to that conclusion.
1. Harmless Error will be an issue.
2. An evidence objection wrongly decided on is looked upon an appellate court by evidentiary
harmless error. As long as there is enough evidence on the record so as to support the holding then it
is ok.
3. Constitution Error- that the error impaired a constitutional right of the D.
7. Questions Must be Specific
8. Questions Assuming Unproved Facts
a. Expert Witnesses - allowed to express their opinions on relevant matters as long as proper
foundation has been laid
A. Validity of Opinion depends on special knowledge not found in lay jurors
B. Witness must be qualified as an expert in the field
C. Must possess a reasonable degree of certainty about her opinion or conclusions
D. Expert must first describe the data on which her conclusion is based
b. Hypothetical Questions Under the Fed. R. of Evidence
A. Rule 703 - expert can base her testimony on facts or data perceived by or made known
to the expert at or before the hearing
B. Rule 705 - expert can testify about her conclusions and explain the reasons for them
without prior in-court disclosure of the underlying facts or data, unless the judge requires
disclosure
c. Laying the Foundation for Admission of Evidence - support for the testimony of witness or the
exhibit they are sponsoring
A. Rule 602 - A witness may not testify to a matter unless evidence is introduced sufficient
to support a finding that the witness has personal knowledge of the matter
d. Using an Interpreter - must swear or affirm that he will truly and correctly translate the questions
of counsel, and the answers of the witness
B. Cross-Examination of Witnesses- Leading questions are allowed
1. Relevance is the true test - cannot range too far beyond opposing counsels direct examination of the
witness
C. Direct vs. Circumstantial Evidence
1. In law, both types of evidence are equal.
2. In front of the jury one type may be stronger than the other.
D. Tangible Evidence - as opposed to testimonial evidence
1. Real Evidence - the real thing
4
Downloaded From OutlineDepot.com

a. Can be direct evidence offered to establish facts about the tangible thing itself
b. Can be circumstantial offered as the basis for an inference that some other fact is true
c. Six Steps
A. Marking for Identification
B. Laying the Necessary Foundation - witnesses sponsor the exhibit and identify and
authenticate and illuminate its relevance to the issues in the case
C. Offering the Exhibit into Evidence
D. Securing an Express Ruling on the Record
E. Precautionary Measure - take out the words for identification to make it clear that it
was received as evidence
F. Showing or Reading of the Exhibit to the Jury
2. Demonstrative Evidence - explanatory or illustrative purposes only; does not go into the jury room for
deliberation
a. Selected demonstrative evidence - standards of comparison
b. Prepared/Reproduced demonstrative evidence - model or diagam
3. Writing - must be authenticated
a. By notice or request to admit genuineness, as under FRCP 36
b. Direct evidence by the author or anyone observing it being made
c. Circumstantially by expert, someone familiar with person in questions handwriting, or letting
jurors compare
E. Judicial Notice
1. Form of evidence, substituting for more elaborate proof of facts that are subject to common knowledge, or
capable of accurate and ready determination by resort to sources whose accuracy is not disputed
a. Must make an on-record request for the taking of judicial notice and for giving of an appropriate
jury instruction
b. Presentation to the court, on the record, of any necessary back-up information.
III. Objections to Evidence
A. Reasons for Forgoing Available Objections
1. Trial counsel does not wish to object to every innocuous leading question since it sometimes helps speed
things up
2. Counsel may let a questionable objection go because of risk that he will only underscore hurtful testimony
3. Does not want to give jurors the impression that he is overly obstructive
4. When answer actually in some way favors his client
5. Offer of objectionable evidence opens the door for more important evidence that silent lawyer hopes to
offer later
B. Time for Objecting
1. Failure to make a timely objection, in proper form, to an offer of evidence will usually operate to waive any
possible basis of complaint about its receipt
2. Objection must be made as soon as the basis for it becomes apparent
C. Objecting to Exhibits
1. Objections before counsel has had a chance to lay foundation are premature
2. Pre-Trial Objections to Exhibits - trial judge will decide on conditional pre-trial evidentiary rulings for the
guidance of counsel
a. Documents admitted conditionally are reoffered during trial, at which time a further ruling, now
unconditional, will be obtained
b. Documents which have been unconditionally excluded at a pre-trial conference need not be
reoffered during trial - still improper
D. Specificity of Objections
1. Counsel seeks to educate the trial judge on the rule or rules of evidence that authorize the objection and the
exclusion of the challenged evidence
2. Counsel is preserving a record for possible appeal in case the judge overrules his objection
3. Counsel is making a record that will support the trial judge on appeal in the event that she sustains
5
Downloaded From OutlineDepot.com

counsels objection
4. Counsel should be explicit about his or her legal grounds - general objection is enough where receipt of the
offered evidence cannot be justified on any legal basis at all
5. Objection must be interposed each time the evidence is offered, unless the trial court permits the objection
as continuing to all related
IV. Offer of Proof
A. Distinguished from Offer of Evidence
1. Offer of proof can come into play before or during an offer of evidence
2. Can also come into play when counsel makes an offer to prove specified matters in order to induce a ruling
by the trial court s to the relevance and competence of those matters
B. Offer of Proof During the Examination of a Witness
1. Used when opposing counsel objects - must then make an offer of proof unless she is prepared to concede
the merit of her opponents objection
2. Permits the trial court to make a fully informed ruling on the objection
3. If the ruling is adverse to the introducing party and arguable erroneous, an adequate offer of proof is
ordinarily essential to preserve the point for post-trial review
4. Three Basic Ways
a. Tangible Offer - hand it to the court reporter for inclusion in the trial record; must state the
evidential purpose of the evidence
b. Witness Offer - counsel proceeds with the examination of question/answer and the recorded
responses (out of jurors hearing) constitute the offer of proof
c. Lawyer Offer - counsels statement of what the witness answer would have been; show that
response would have benefited client
C. Offer of Proof with No Witness on the Stand
1. Can be made to induce a ruling with respect to a line of facts - allows a court to pre-test proposed evidence
and avoid possible prejudicing of the jurors (out of hearing of jurors)
2. Must include specifics - not enough to be general - and must be made in good faith on the basis of evidence
that is known to be available and beneficial
3. Offers of proof in complicated cases are usually required to be in writing
a. Identification of the expert
b. Summary of his qualifications
c. Detailed disclosure of the factual data and scientific, technical, or economic authorities and other
material relied on by the expert in arriving at his or her opinion
d. Clear statement of the experts opinion
e. Summary of the reasons for the opinion
D. Renewing Offers of Proof
1. Offers can be renewed if they are originally defective and have now been perfected
2. No need to renew when the court has made clear that under no circumstances will they receive the evidence
E. Making Offers Outside the Jurors Hearing
1. Trial courts usually require offers to be made out of jurors hearing because of the potential for prejudice
V. Instructions to the Jury
A. Judge will either require or at least permit participation by counsel in the preparation of the courts charge to the jury
1. Counsel will have researched and briefed the underlying law and drafted the proposed instructions prior to
the commencement of the trial
2. Conference is held after the parties have rested and before closing arguments commence
3. Objections must take place out of the jurors hearing immediate after judge has given the charge, otherwise
objections are waived
a. Objections must be specific to give the judge an opportunity to correct any errors that he can be
convinced he made
VI. Verdicts
B. Special Verdict - jury makes specific written findings on each fact-issue in the case; court gives the jurors written
questions (Yes or No)
6
Downloaded From OutlineDepot.com

1. Counsel can make a specific demand, on the record, that the trial court submit a particular fact-issue to the
jury
C. General Verdict - jury simply finds for the plaintiff or defendant, and any damages

---------------------------------------------------------------------------------------------------------------------
CLASS NOTES - 8/19
$ The Adversary System - clash of proofs is likely to result in information which a neutral and passive decisionmaker can
base a resolution acceptable to both parties

$ Fed. R. Evid. 103 - lawyers have the responsibility of presenting evidence and raising
objections. Failure to object constitutes a waiver of the issue.
$ The Exclusionary Rules of Evidence - Rule 103 states that objections must be timely and specific. If
evidence is excluded, counsel must make an offer of proof to preserve the issue for review.
$ Federal Rules of Evidence are designed to give the judge guidelines on making rulings to
objections/admissions, etc.

$ Neutral and passive decisionmaker - rules of evidence are applied by the trial judge. Appellate courts defer to the discretion
of the trial judges ruling on evidentiary issues

$ Jury - the exclusionary rules of evidence reflect societys commitment to vest in lay citizens the major responsibility in
deciding disputed cases

CHAPTER ONE
I. Relevancy
A. Thayers Definition
1. Nothing is to be received which is not logically probative of some matter requiring to be proved
2. Everything which is thus probative should come in, unless a clear ground of policy or law excludes it
B. Union Paint and Varish Co. v. Dean- Two drums of paint. P returned a can a paint and said that it was defective
because he said that the other can of paint that he had was defective.
1. The other can of paint does have a tendency to prove a fact of consequence because it was made from the
same lot of paint.
C. Knapp v. State- Police Officer shot by D who admitted it. D claims self-defense claiming that he had heard that the
officer had killed another person.
1. D must prove a reasonable belief that the officer was going to kill him and that killing the D would
have been a reasonable amount of force to defend himself.
2. State calls the coroner who testifies that the other old man died of alcohol
3. D objects arguing that the relevant fact was Ds reasonable belief and not the cause of older mans death
4. Fact of consequence that was relevant was the likelihood that D heard the rumor that the old man was
killed by the officer. Because this fact was not true, it was less likely that D actually heard this rumor.
Therefore, the fact of the old mans actual death tended to disprove the fact that D heard this rumor.
D. Relevancy is not an inherent characteristic of any item of evidence but exists as a relation between an item of
evidence and a proposition sought to be proved
1. Item that tends to prove or disprove any proposition, is relevant to that proposition
E. Rule 401 - Defines relevancy-
1. 2 Step Analysis
a. Offer of evidence must have ANY tend to prove a fact
b. Must be a fact of consequence (must be provable in the case)
i. Initially what is a fact of consequence is determined by the pleadings - the elements in the
cause of action and defense
ii. What is a fact of consequence can expand as parties develop their case
2. 401 encourages a relevancy finding
3. Initially, a test of common sense logic - certainly not a test of formal logic
7
Downloaded From OutlineDepot.com

4. Dont confuse relevancy admissibility analysis with the directed verdict sufficiency analysis. No one offer
of evidence need prove a fact, only slightly alter the probability of the fact.
F. Rule 402 - Rule of Relevancy
1. All relevant evidence is admissible unless otherwise excluded. E.g. exclusionary rule, a statute, a rule of
procedure, or the constitution.
2. Evidence that is not relevant is not admissible.
II.Introduction to Relevance
A.Evidence must be material and probative
1.Material = offered to prove a properly provable issue in the case (what is the evidence trying to show and is this an issue in the case)
2.Probative = It must logically tend to prove the proposition for which it is offered
B.Two reasons offered items of evidence may be excluded as irrelevant:
1.It is not probative of the proposition at which it is directed
2.Because that proposition is not provable in the case
C.When figuring out if something is relevant, ask three questions
1.What is the evidence being used to prove?
2.Is that proposition provable?
3.Does the evidence help?
D.401 Relevance Definition
E.402 Relevant evidence generally admissible, irrelevant evidence inadmissible
F.403 Exclusion of relevant evidence on grounds of prejudice, confusion, or waste of time
G.Inference
1.A brick is not a wall not every piece of evidence has to be a homerun
2.Besides being material, relevant evidence must be probative
a.Probative = evidence that logically tends to prove the proposition for which it is offered
3.(Knapp v. State) Evidence is relevant if it tends to support an inference that goes to a central issue of the case
a.If even a slight inference can be made, the court should admit the evidence of a collateral fact.
b.Ct. must balance probative value v. prejudicial value
4.(Old Chief v. U.S.) It is an abuse of discretion to admit the full record of a prior conviction when the accused is willing to
admit the offense for the purpose needed
a.FRE 403 allows exclusion of relevant evidence when probative value is substantially outweighed by the danger of unfair prejudice
i.Unfair prejudice = an undue tendency to suggest decision on an improper basis
b.FRE 404 specifically prohibits the admission of evidence of other crimes to prove the character of a person in order to show
conformity
5.(Ballou v. Henri Studios) 403 permits excluding evidence if its probative value is substantially outweighed by the danger
of unfair prejudice
a.Unfair prejudice in 403 means an undue tendency to suggest a decision on a improper basis, not necessarily an emotional one
b.Weighing probative value against unfair prejudice = the probative value w/ respect to a material fact if the evidence is believable,
NOT the degree the court finds it believable
III. Probative Value vs. Prejudicial Effect
A. Rule 403 Balancing Test
1. Relevant evidence maybe excluded if its probative value (its tendency to prove a fact of consequence- or
alter the probability of that fact) is SUBSTANTIALLY (meaning when the judge says I agree)
outweighed by (danger that it is doing it unfairly)...
a. Unfair prejudice, confusion of issues or misleading the jury; or
b. Undue delay, waste of time or needless presentation of cumulative evidence
B. Old Chief v. United States Assault and Felony in Possession of a Weapon; Prior was Assault w/ Deadly Weapon-
objection regarding prior felony was unfair prejudice. Section 1 from pg. 84-86 is the most important section in
the case.
1. Issue: Whether a district court abuses its discretion if it rejects an offer for an admission, and then admits
the full record of a prior conviction when the name and nature of the offense raises the risk of a verdict
tainted by improper considerations
2. Court held that court did abuse its discretion because the stipulation as to the conviction was offered and by
8
Downloaded From OutlineDepot.com

offering the name and nature of the prior offense, it worked more as a proof of bad character. Telling the
jury of the prior felony that was the same as the present charge might go towards showing a prior bad act
which would make it more likely that he committed this crime.
a. Unfair prejudice - concededly relevant evidence that lures the factfinder into declaring guilt on a
ground different from proof specific to the offense charged
b. Although counsel is allowed to prove its case by evidence of its own choice, the risk that a jury
will convict for crimes other than those charged creates a prejudicial effect that outweighs
ordinary relevance
c. The offered stipulation that D is solely a felony without information as to what the felony was is
more probative then the existence or admittance of the judgement and order of the prior felony
which states what the felony was.
d. Souter says allows the more probative and less prejudicial stipulation be the judge must examine
the narrative richness of the item
i. Judge must evaluate the narrative richness of the item independently. Professor Cribar1
likes Souter!
ii. The event or circumstances of the prior felony was irrelevant in this case. It was a status
offense. The prior felony had no narrative richness to it.
3. The Island Comparison Test:
a. Island: What is the Evidence Offered to Prove?
i. Look at the evidence by itself
b. Comparison: Look at the Evidence to determine if the evidence is unfairly prejudicial when
compared to other crimes charged or other elements of the crime.
C. Ballou v. Henri Studios, Inc.- Ballou died as a alleged result of the negligence of D an employee of Henri Studios.
(driver of a truck)
1. Trial Court cannot exclude evidence on the basis of the veracity or his or her belief of the evidence.
2. The judge has to assume that the evidence is true
D.Probabilistic Evidence
1.When a certain event cannot be proved by direct evidence, parties resort to proof based on probabilities. This evidence is not
favored despite the standard of proof in a civil case being more likely than not. Cts require more certainty than a mere statistical
livelihood
2.(People v. Collins) Mathematical proof generally can only be used to support already adequate proof, cant by itself be
used as a foundation in criminal cases
a.This type of testimony can distract the jury from its proper function of weighing the evidence on the issue of guilt
3.(People v. Mountain) Evidence of a Ds and the assailants blood types are admissible in sexual crimes cases
a.When identity is at issue, proof that a D and the perpetrator share a relevant physical characteristic, such as blood type, does not lack
probative value b/c a large number of other people share the same characteristic
(Kammer v. Young) Expert opinion evidence based on statistical probabilities can be used to prove paternity
CHAPTER TWO
I. Hearsay- Rule 801- Non Hearsay is admitted because it is reliable.
A. A Statement: An oral or written Assertion or nonverbal conduct of a person, if it intended by the person as an
assertion.
B. An Assertion: An intention to communicate something
C. Declarant: person making the assertion (referring to another person who is not in the courtroom)
1. Could have a problem when declarant is the witness. Cant testify to your own hearsay. It may be
admissible but it is still hearsay. There may be an exception that would allow it to be admissible, but it is still
hearsay.
D. Rule 801(c) Hearsay is.... Focus on the statement not on the maker of the statement or his
or her intention. An Assertion-centered analysis- offered in evidence to prove the truth of the matter
asserted. A declarant-centered analysis (which this course is not based) depends on the credibility of the
declarant.
1. Out of court statement

9
Downloaded From OutlineDepot.com

2. Offered in court
3. Offered to prove the truth of the matter asserted (sometimes to prove what another
witness said)
i. highly technical
ii. Much hearsay is admitted at trial
iii. Hearsay rule is one of several rules of exclusion
iv. The hearsay exceptions are not rules of admission (Rule 803, 804)
E. Hearsay ConcernsHearsay is presumptively unreliable. Therefore it is presumptively
inadmissible. Even if reliable not necessarily admissible. It has to fit specifically into one of the
exceptions. .
1. Ambiguity
2. Sincerity
3. Memory
4. Perception
5. Others might add avoiding surprise, avoiding sharp practices and promoting fairness
F. Trustworthiness at trial is protected by...
1. Oath
2. Cross-examination
3. Jurors can see demeanor
4. No chance of mistransmission
G. Out of Court Statement
1. Witnesses can testify to what they saw
a. Witness may testify I saw the accident. The red car ran the light.
2. But when they repeat statements, there is a hearsay problem
a. If witness testifies Immediately after the accident, I said (or Bob said) The red car ran the light
we have a potential hearsay issue
3. Oral Statement what witness or someone else said
Non-verbal Statement conduct intended by declarant to be an assertion
a. Nodding the head, pointing, finger conversation
H. Truth of the Matter Asserted
1. What is the message advanced by the assertion?
2. Why is the assertion being admitted?
3. Example Witness testifies that mechanic said You have bad brakes.
a. Hearsay if admitted to prove the brakes are bad
b. Not hearsay if it was offered to prove other facts such as...
i. Owner was put on notice of the possibility that brakes might be bad
ii. The mechanic can talk still must be relevant
iii. The mechanic has met the owner
iv. Statement offered to impeach the mechanic because
in court he testified that the brakes were fine
I. State v. English- Confession of a 3rd Party not admissible. The words of a stranger to the parties and not spoken on
oath violate the protections of with which hearsay protect. (There will be a hearsay exception for this later, but it is
still hearsay!!!)
1. Even a judgment and commitment form is hearsay because it merely memorializes what the judge said at
the other trial.
2. Even a statement made in another courtroom is hearsay and there is no exception for statements made in
other courtrooms
J. Tribe, Triangulating Hearsay- Concerns
1. Ambiguity
2. Insincerity
3. Erroneous Memory
4. Faulty Perception (ability to perceive)
10
Downloaded From OutlineDepot.com

F. Non-Hearsay Statements Offered to Prove...


1. Rule 803(3) - Physical or Mental State of Declarant
a. He was alive, under duress, or confused about something
i. Listeners state of mind must be relevant when the
witness hears something or something is said to them
b. State of mind must be a fact of consequence / relevant
c. Once you prove that it shows someones state of mind, you
must then show why it is relevant
d. Murdock I am alive Statement proves he could talk and therefore
was alive. The act of statement itself is not hearsay. If it is admitted to prove the truth of the
statement then it would be hearsay.
Subramaniam Possession of Ammunition charge duress by
Terrorists. Statement of terrorists admitted. Statement proves the effect on the listener he was
under duress and had a reasonable fear regardless of whether they
Vinyard Parking Lot slick/complaints testimony. Statement
of complaints goes to prove the effect did the defendant have knowledge that the parking lot
might be slippery (even if the statements werent true)? Not admissible to show parking lot was
slippery when plaintiff fell
Johnson v. Misericordia Records of doctor admissible. Out
of court statement that Salinsky was a bad doctor also showed to prove hospital was on notice
Ries Biologicals, Inc. v. Bank of Santa Fe- Debt of Bus. In
kidney dialysis and guarantee of bank is admissible. Statement is offered to prove that the bus.
Was reasonable in relying on statement to resum sending the goods.
3. Legally Operative Facts: Verbal Acts - words of legal significance are not precluded under hearsay. They
are not hearsay. Common Law Doctrine applies and the rules dont apply, so these statements are not
hearsay.
a. Words on orally entering into a contract or negotiations
b. Giving a gift + words. Vows of Marriage.
4. Fun-Damental too, Ltd. V. Gemmy Industries Corp. Toilet Bank Trade Dress Claim. Testimony was
offered to prove that there was confusion in the industry
5. U.S. v. Hernandez- DEA referral from customs testimony not admissible. It showed how the DEA became
aware of Hernandez. The effect on the DEA agent. But the objection should have been sustained on
relevancy grounds.
6. Silver v. NY Cent. RR- Pass. Was cold and there were 11 other passengers. Porter on the stand could not
testify to other passenger communications regarding tempature of car. Not saying something asserts that
the passengers thought the train was not cold. The fact that the 11 other pass. Did not complain ould be
seen ashearsay as it asserts that the other passengers thought the tempature was ok.
a. Rule- The absence of a complaint or statement will not be identified as hearsay.
7. U.S. v. Jaramillo-Suarez- Pay/Owe drug sheets admissible as circumstantial evidence. Ledgers were
offered to prove through circumstantial evidence to show the character and use of the place where they
were found.
8. Computers- Is it computer generated date or computer stored data? Important because only a person can
make a hearsay statement.
9. U.S. v. Brown- IRS Audit testimony inadmissible as based on conversations out of court. IRS agent
testified to a conclusion. It was not testimony that asserted the statement to prove the fact of the matter
asserted. The better opinion in this case is the dissenting opinion. This is not a hearsay problem. No
statement was testified to by the IRS agent. She was an expert (or could have seen to have been one).
10. City of Webster Groves v. Quick- Violation of speed ordinance. Reading of the mechanism is not hearsay
because the radar gun is not a person. If second officer testified it would be multiple hearsay. Most state
legislators have a legislative enactment that say this is not hearsay in state court.
10. Conduct
a. Conduct that is intended as an assertion (trying to
11
Downloaded From OutlineDepot.com

communicate a message pointing, nodding) is treated as a statement


b. Non-assertive conduct opening an umbrella, driving through
an intersection, etc. is not a statement
11. Words might be offered not to prove an assertion but as circumstantial
evidence of other facts of consequence
a. A mug with Peters name on it in the room might go to show that Peter was in the room
G. Hearsay
1. Hearsay rule will overlap with other evidentiary rules, such as Rule 602 requiring personal knowledge and
Article 7 regulating expert testimony
2. Rule 801 requires statement be made by a person. Statements made by
parrots, machines, watches or dogs are not hearsay statements
a. May be excluded under relevancy principles, but theyre not hearsay
H. 3 categories of statements that appear to be hearsay but are not
1. Exclusions Rule 801(d) Prior Statements of witness if...
a. Inconsistent and the prior statement was under oath
b. Consistent and rebuts an express or implied assertion of
recent fabrication or improper notice
a. Implied Assertions- U.S. v. Zenni- Testimony of Telephone Conversation re: betting.
Implied Assertion not hearsay. Statements that dont assert anything cannot be hearsay.
What is being implied is that this residence is an illegal betting parlor. This is allowed
because the statement doesnt assert that this residence is a betting parlor. Whats implied
is not a statement-
MN Sup. Ct. Case- A061743- U.S. v. Her- (READ AFTER CRAWFORD AND DAVIS FROM
SUPPLEMENT
THE HEARSAY RULE
I. Rationale and Meanings
a. FRE 801 (c) Hearsay is a statement, other than one made by the declarant while testifying at trial/hearing,
offered in evidence to prove the matter asserted.
b. Rationale Reflects concern about trustworthiness of hearsay evidence. Evidence is questionable b/c not given
under oath, or where fact finder could observe declarants demeanor, and not subject to cross-x by opposing counsel
to test perception, memory, veracity of the actor
c. (State v. English) Evidence of confessions by 3rd parties in criminal cases is excluded
i. Not all evidence excluded by the hearsay rule is bad evidence
d. (Estate of Murdock) If evidence of an out-of-court assertion is used not to prove the truth of the matter
asserted but only to show the assertion was made, it is not hearsay
i. Case where plane crash victim says, Im still alive
ii. 801 (a) defines a statement as an oral or written assertion, or a non-verbal conduct of a person, IF it is
intended as an assertion
1. Here, the statement was an assertion, the case is based more on the fact that Murdock said
something/anything, not that he claimed to be alive
e. (Subramaniam v. Public Prosecutor) Statements are allowed when they are not introduced to show the
truth of statements made, but to show that statements were made
i. If evidence is admitted to show its effect on state of mind and not the truth of any issue, it is not hearsay
f. (Vineyard v. Vineyard Funeral Home) Hearsay rule does not apply when the fact that a statement was made
is relevant, regardless of whether the statement is true or false
i. The hearsay use would be if the statements were used to prove ramps were slippery, but they were used to
prove that agents of the funeral home had prior knowledge of the danger posed by the ramp.
g. (Johnson v. Misericordia Hospital) Evidence not admitted to show the truth of opinions but to show that
opinions exist is not hearsay
i. Records of opinions may be admitted to show the ready availability of the opinions in the records, even
when the opinions bear on the ultimate issue of the case
h. Legally Operative Facts
12
Downloaded From OutlineDepot.com

i. When the issue revolves around words that are legally significant in and of themselves, regardless of their
veracity, evidence of the words is admissible
i. (Ries Biologicals v. Bank of Santa Fe) The relevance of out-of-court statements depends if they are
admitted for a non-hearsay purpose (the fact the statements were made, not the truth of falsity of the
statements)
i. Testimony can be admitted for the sole purpose of showing the statement was made
j. (Fun-Damental Too v. Gemmy Ind.) Hearsay evidence can be admitted if it is not used to prove the truth of
the matter asserted, but simply probative of the situation
i. FRE 803 (3) statements otherwise excluded as hearsay may be received to show the declarants
then-existing state of mind
k. (U.S. v. Hernandez) Hearsay evidence may not be admitted to show something or be used for a purpose not
at issue in the trial
l. Non-Assertive Conduct
i. Is generally admissible from the federal rules, not hearsay by virtue of its definition. Differs from common
law
ii. Implied assertions do not share the dangers of hearsay
m. (U.S. v. Zenni) Out-of-court statements by absent declarants can be admitted as non-hearsay when offered
solely to show declarants belief in a fact sought to be proved
i. The utterances of the gamblers who called in bets were non-assertive verbal conduct, offered for an implied
assertion that bets could be placed at the premises called
1. The callers did not intend to make an assertion when they spoke, thus admissible
n. (Commonwealth v. Knapp) Webster argues that suicide can be viewed as a confession, evidence of a guilty
conscience
i. The alleged killers suicide was a non-assertive conduct revealing that the killer had a guilty conscience
from which death was the only escape
ii. Suicide as evidence of state of mind
o. (Silver v. NY Central RR) Negative evidence (uniform silence) on the part of many is probative evidence
and allowed if the circumstances of P and others are the same
i. It is a reasonable inference to make that if no one complained, no one suffered
ii. Silence is the basis of an affirmative inference of proper conduct or conditions
iii. The hearsay problem is generally ignored in silence cases, the focus is on relevancy
p. (U.S. v. Jaramillo-Suarez) Drug-related documents may be admitted to prove the character and use of the
place where they are found
i. It has probative value for the purpose it was admitted and was independent of the truth of its contents,
hearsay not implicated, it is allowed
q. (U.S. v. Rhodes) A declarants statements indicating belief may be admitted as evidence of the truth of that
belief
i. The purpose was not to prove the assertions it contained, but to demonstrate an inordinate interest in
Rhodes by the Russians
r. (U.S. v. Brown) Testimony is hearsay when it is based on the witnesss knowledge gained from statements
of out-of-court declarants
i. Since testimony was based on out-of-court statements, D was deprived the opportunity to test the
assumptions through cross-x.
ii. FRE 701 opinion testimony by lay witnesses (rule requiring 1st hand info)
s. Non-human evidence
i. Testimony by a witness as to statements made by non-human declarants, such as instruments, is not
hearsaynon-human sources lack a conscious motivation to lie
t. (Webster Groves v. Quick) Testimony concerning observations and perceptions obtained through the use of
scientific instruments is not hearsay
i. The guarantee of trustworthiness is satisfied by the exercise of the right of cross-x of the witness on the
stand, both as to the results obtained and his testimony as to the reliability and accuracy of the device used

13
Downloaded From OutlineDepot.com

Statement Under Belief of Impending Death- Dying Declarations- Common Law: Statements someone said at the point of death.
Something declarant says when she believes that she is at the point of dying. Exception because they meet the requirement of
reliability. Not an execption to the definition of hearsay just an exception to hearsays admissibility
Reasoning: Scalia-Who would want to meet their make with a lie on their lips. This might be an exception to the
6th Amendment (Confrontation Clause).
c. Dying Declaration Rule 804(b)(2)
i. Declarant must be unavailable- 804 a gives examples of
unavailability- Proponent of the hearsay must make a showing that
there is a good faith effort to produce the declarant.
ii. Must be charged w/ Homicide in a criminal case or civil action-
Unknown whether there is a requirement that it must be the declarants
homicide for which D is on trial. Some courts use 403 to avoid this
(prejudicial).
iii. Made while believing death was imminent
iv. Concerning cause or circumstances of
impending death
1. Personal Knowledge needed
2. Judge must find preliminary facts Rule 104(a)
3. Judge may consider non-admissible evidence- does not have
to follow rules of evidence in making this determination
whether it is a dying declaration. Privileges always apply
though!!!
4. Judge must find facts by preponderance of evidence- Once
judge made a decision regarding admissibility the question
becomes moot.
I. Dying Declarations
a. A victims dying declaration about the cause of her imminent death is admissible under FRE 804 (b) (2). The fear
of death supplies an intrinsic sufficient trustworthiness.
b. Admissible in all civil cases but no criminal cases except homicide
c. The dying declaration must satisfy the following requirements: (pros. has burden of est.)
i. Victim as declarant
1. Declaration must be made by the victim
2. Does not apply to the deathbed confession of a person who claims to have killed the victim
ii. Sense of imminent death
1. The declaration must be made while actually believing death was imminent
iii. Scope of declaration
1. The declaration must state facts about the cause or circumstances of the victims impending death.
An opinion does NOT fall within the exception
iv. Death requirement
1. Traditionally, the exception only applied if the declarant had actually died by the time the evidence
was offered
2. As long as the declarant had a sense of imminent death when the statement was made, it did not
matter how much time elapsed b/w statement and actual death
3. However, the victim does NOT need to be dead at the time of the trial, if she is otherwise
unavailable, so long as she believed death was impending 804(b)(2)
d. (Soles v. State) A judge must decide if a statement offered as a dying declaration was actually made w/ the
consciousness of impending death, a jury can only decide the weight to be given the evidence after it is
admitted
D convicted of manslaughter. V states to father that D shot her and death was imminent. You may
consider the statement as a prereq. That the requirement of belief of impending death was present.
Basically, it is admitted as a declaration or not, it cant be admitted, then a jury be
instructed that they can exclude it from consideration
14
Downloaded From OutlineDepot.com

Dying Declaration is a legal creature. You only have one when there is a trial in a homicide trial or a civil case.
Judge must leave the question of whether to believe the testimony or not to the jury. If both statements could be true then
the judge should try to admit them both and let the jury decide which to believe.
Present Sense Impression- Admitted because there is no time to reason about the statement. No time for fabricating because the
statement is temporally linked.
a. Present Sense Impressions -- Rule 803 (1)
i. Statement describing or explaining an event or condition made while the declarant was perceiving the event
or condition, or immediately thereafter
d. Excited Utterance Rule 803(2)- There may be time to reason, but emotions overwhelm her and
cause the statement without regard for the amount of time that has passed.
i. Hearsay statement relating to a startling event made while
declarant is under stress from the event
ii. Judge can disregard the FRE if he deems if necessary, except for the rules of privilege
iii. Well established exception
iv. Some showing of personal knowledge required
1. Judge determines whether the elements are established under 104(a)
2. Judge can consider the statement itself bootstraps
3. No corroboration requirement
c. Spontaneous & Contemporaneous Exclamations
b. A statement by any person made in direct response to an exciting event may be admissible under FRE 803(2)
c. The requirements of this exception are:
i. Startling Event
1. The occurrence producing the exclamation must be startling enough to produce shock, excitement,
or a similar reaction in the observer
ii. Spontaneity
1. Must be spontaneous, made while under the influence of the shock or excitement. It must be made
contemporaneously with the event
iii. Scope of Statement
1. The exclamation must relate to the exciting event that provoked it. Many courts will not allow
statements of opinion, particularly when the fix blame
d. (Truck Ins. Exchange v. Michling) For an excited utterance to be used as an exception, there must be
independent evidence of the occurrence giving rise to the utterance (evidence of startling event is absence,
temporal link is not present).
i. A statement concerning a traumatic injury( hit head in a bulldozer), made by an out-of-court declarant on
the same day the declarant suffered the injury to wife re injury but some time after the injury occurred may
NOT be admitted as evidence to prove the source of the injury. Statement was not made while precieving
the event or immediately thereafter. This is hugely prejudice. (403 issue also).
e. (Lira v. Albert Einstein Med) A non-testifying physicians surprise at medical condition is not an excited
utterance (or present sense impression)this exception requires spontaneity

i. Whos the butcher who do this? said the doctor. He could have qualified as an expert witness, but he
didnt come to court. So they tried to testify as to what doctor said.
ii. The exception applies to a spontaneous declaration by someone affected by an overpowering emotion
triggered by an unexpected and shocking event
iii. If Lira had been decided under the Federal Rules, different result?
1. Excited utterance? Probably not, he was a doctor (specialist even)
2. Was the event really stressful to the declarant?
3. Present sense impression FRE, statement has to describe a condition observed- the statement
was probably an opinion as to the prior surgery.
f. (State v. Jones) Contemporaneous statements of present sense impressions by unknown declarants can be
admitted as evidence
i. MD state police assault case. 2 statement over CB radio by truckers.
15
Downloaded From OutlineDepot.com

ii. A statement of present sense impression must be essentially at the same time with the event it describes
iii. The declarant must speak from personal knowledge, but his identity need not be proved when the statement
itself demonstrates the percipiency of the declarant
iv. The statement itself authenticates itself.
v. Practice Point: When you win a hearsay objection only those words can come into evidence and nothng
else. Testimony only to what he heard exactly. Not to conclusions made or a narrative.
Exception - Availability of Declarant Immaterial Rule 803
a. Present Sense Impression Rule 803(1)
i. Hearsay statement describing or explaining an event or condition made while declarant perceiving or immediately
thereafter
1. Not a traditional hearsay exception
2. Must be contemporaneous
3. Personal knowledge required
a. Court may conclude there is personal knowledge from context of the statement Rule
104(a)
b. Compare Look at that car speeding down the street. vs. There was a car speeding
down the street.
Admissions-
d. Party admission - Rule 801(d)(2)
a. Not Hearsay- Statements by a party that are contrary to the partys litigating posture are
excluded from the definition of hearsay- Offered against the party and is the partys own statement.
i. Although there may be some indicia of reliability people usually do not say
things that might go against their interests party admissions come in as a result
of the adversary system
ii. Party admissions are readily admissible even if indicia of reliability are lacking
iii. Party admissions are admissible even if the party lacked personal knowledge
iv. even if it is in opinion form
v. Contrast party admissions with the limited exception
for Declaration Against Interest (804b3)
b. Statement by a party
i. confessions
ii. plea of guilty
iii. statement to friends
iv. statement to insurance company
c. Statement by an Agent Authorized to Speak
i. lawyers
ii. public relations
iii. some executives in corporations
d. Statement by an Employee Concerning a Matter within the Scope of Employment
i. Under the federal rules, party admissions are not limited to statements by
speaking agents
ii. Statements made concerning the scope of his authority, while still an employee,
are party admissions
iii. Cannot talk about something related to the company, which is outside your
scope of employment
iv. The statement of the employer is not a party admission against the employee

Adopted Admissions
i. One can expressly adopt anothers statement
ii. A party could say I agree with that
iii. A party could sign a statement prepared by another
iv. A statement adopted by a party can be a party admission
16
Downloaded From OutlineDepot.com

v. Implied Adoptions
1. Difficult issues
2. Express agreement by nodding or
remaining silent when a reasonable person would speak if they
disagreed
3. Silence can be taken as acquiescence or agreement
vi. State v. Carlson
1. To prove possession of controlled substance, state offered testimony
that Lisa said You liar, you got those tracks shooing up in your room
with your friends.
2. If offered as Lisas statement = hearsay, OK if offered as Ds statement
b/c he has adopted it by hanging his head and not refuting it
a. Silence can be misconstrued as acquiescence when there are
other explanations
b. Did they hear and understand it? Free from intimidation that
would impede speaking up?
c. In deciding whether silence should be treated as adopting a
statement, the judge should use Rule 104(a) decide by a
preponderance of evidence (more likely than not) that D
intended to adopt this statement
d. If it were a 104(b), issue would be sent to the jury if there was
some evidence that jury could find that the statement was
adopted
II. Admissions
a. Any out-of-court words or acts of a party that are inconsistent w/ the position that the party takes in the current
proceedings may be offered to show the truth of the matters asserted in the out-of-court statement
b. Not referred to as an exception to the hearsay rule, referred to as an exemption to the rule
c. Admission does not have to be admitting something or be against his interests
d. General requirements
i. Unavailability not required
1. the party against whom an admission is offered need not be unavailable
ii. Personal knowledge
1. Most courts do not require that the hearsay declarant (the party against whom the admission is
entered) have personal knowledge of the facts admitted
e. Admissions by Conduct
i. Adoptive admissions by conduct
ii. Circumstantial manifestations of assent by conduct
iii. Adoptive admissions by silence (tacit/implied)
iv. Would a reasonable person in a similar situation have denied the statement?
1. The reaction of the D is the keythat constitutes admission by conduct
f. (Reed v. McCord) Admissions do not have to be based on first-hand knowledge to be admissible (civil or
criminal)
i. It is highly improbable that a party will admit or state anything against himself or against his own interest
unless it is true
g. (U.S. v. Hoosier) Statement may be offered as an admission by silence, D silence constitutes an adoption of
the declarants statement- Adoption by silence
i. Girlfriends statement regarding money and D was silent (implication that money came from bank
robbery). Operates to make her statement his admission.
ii. More is required than mere presence and silence of the party against whom the admission is sought
iii. Rule: You should protest if a statement like this is made.
iv. Standard of proof is the preponderance of evidence

17
Downloaded From OutlineDepot.com

h. (State v. Carlson) A judge may make a finding of fact (preliminary fact) regarding whether an accuseds
non-verbal reaction to an accusation was accompanied by intent to approve of accusation- Adoption by
Conduct
i. Meth testimony of wife with police officer- Intent to adopt, agree or approve- Not admissible.
ii. Rule: Conduct can serve to adopt, agree or approve,
iii. Admissibility is a question for the judge and the weight and credibility given it to the evidence is left to the
jury.
iv. Manifestation of an adoption may be either a preliminary question of fact for the trial judge under FRE
104(a) or a question of conditional relevancy 104(b)
v. The trial judge has the duty to make prelim determination regarding the admissibility of evidence and intent
to adopt is a requirement for admissibility
vi. Whether the intent to adopt was a preliminary question of fact or was it a question of conditional relevancy
1. If it was a matter of conditional relevance, it goes to the jury
2. If it is a preliminary question, then a judge decides based on pp of e
i. (Mahlandt v. Wild Canid Research) To introduce an agents admissions, the proponent need not show that
the declarant had personal knowledge of facts underlying the statement- Agency Argument.
i. Wolf Mauled Child- Note (801d2d), Statement that wolf bit the child/statement of Poos on the phone
stating wolf bit the child (801d2a), and minutes of Wildlife organization (801d2a for Wildlife Org.) are
admissible.
ii. Requirement that admissions based only on personal knowledge of declarant is unjustified when there is
sufficient prejudice against the declarant in making an admission that declaration is trustworthy enough to
get around the hearsay rule
iii. Why are admissions by agents admissible?
1. For policy reasons, efficiency, the agent is better equipped to know the situations that the
employer in many situations
j. (Big Mack Trucking v. Dickerson) To be admissible against the master, an out-of-court admission by the
agent must be authorized by the master- Glossed over in class.
i. Truck Accident Admissions Death- Statements of D not admissible
ii. State case, Pre-Federal Rules of Evidence Case.
iii. If there was a valid reason why the master should lose the protection of the hearsay rule, it would
undermine the right to cross-x
k. (Sabel v. Mead Johnson) Statements by participants at a meeting convened by a party are not admissible
even if the participants were not employees or agents of the party
i. Mead Medication defect causes priaprism. Outside researchers are not agents.
ii. P has the burden to demonstrate the existence of an agency relationship
iii. Test of Agency:
iv. Power of the agent to alter legal relationships between the principal and third parties and the
principal
v. Existence of fiduciary relationship towards principal with respect to matters within the scope
of agency
vi. The right of the principal to control the agents conduct within the scope of agency.
Co-Conspirator Statements- Statements made by co-conspriator are admissible if made during the conspiracy and in furtherance of
the conspiracy. Admissible against each co-conspirator as if it was the con-conspirators statement.
f. Co-conspirator statement made:
i. Conspiracy exists
ii. Declarant and defendant are part of it
iii. During course of
iv. In furtherance of
1. Proponent must establish each of the elements by a preponderance of
the evidence (Rule 104a)
2. Trial judge may consider the statement in determining whether the
elements have been established (bootstrapping allowed)
18
Downloaded From OutlineDepot.com

3. The statement by itself cannot establish the elements of the exception


(partial bootstrapping)
g. Common Law: Standard of proof clear and convincing evidence.
h. Federal Rules: Judge did not need to heed to the Rules of Evidence in determining
admissibility (with the exception of privileges).Standard of proof is preponderance of the
evidence.
2.

III. Admissions: Co-Conspirators


a. Admissions made by a co-conspirator may be introduced against another co-conspirator as an exception to the
hearsay rule.
b. For the admission to be admitted, the following requirements must be met:
i. The conspiracy itself is established prima facie by independent evidence
ii. The statement itself was made during the conspiracy (before the crime was consummated or the declarant
withdrew from the conspiracy); and
iii. The statement was made in furtherance of the conspiracy (it relates to the effort to accomplish the illegal
objective)
c. (U.S. v. DiDomenico) Conspirators are agents/principals of each other, thus bound by their words and
deeds, but limited to the specific conspiracy (n/a to criminal conspiracy)
i. An admission by one is an admission by all
d. (U.S. v. Goldberg) A late-joining conspirator takes the conspiracy as he finds it and assumes conspirators
responsibility for the existing acts of the conspiracy
e. (U.S. v. Doerr) Statements by co-conspirators must be in furtherance of the conspiracy to qualify for a
hearsay exception
i. Strip Club- Co-conspirators
ii. In furtherance can take many forms, but narrative declarations and mere idle chatter are not statements
in furtherance of the conspiracy
f. (Bourjaily v. U.S.) Court may consider statements themselves proof of a conspiracy when determining
whether conspiracy existed to allow statements involving an accused
i. Conspiracy to distribute cocaine- Admittance of co-conspirators statements judge may look to hearsay itself
to prove existence of a conspiracy and Ds involvement in it.- This is a preliminary question for the judge to
answer.
ii. A court must determine as a preliminary question that the statement falls within the rule before it is
admitted
Former Testimony Rule 804(b)(1)
i. Testimony must be made at another hearing or trial or deposition
ii. In criminal matter, accused or prosecutor must have had the opportunity and similar motive to test the witness through
cross-examination
1. The party with the opportunity to cross examine at the prior hearing must have had a similar
motive to fully challenge the testimony
2. In Wright, JBs interest and motive to cross examine the witness to defend his arson trial was
similar to his interest in the civil insurance trial
3. In a criminal case, it must be the same party
iii. In a civil matter, the party or a predecessor in interest (person in privity) must have had the opportunity and similar
motive
1. In a civil case, the party with the chance to cross examine at the prior hearing must be a
predecessor in interest to the party in the instant case
2. Predecessor in Interest suggests privity (grantor/grantee, donor/donee, lessor/lessee, etc.)
I. Former Testimony
a. Transcripts of testimony given by a witness at a former deposition, hearing, or trial, in the same or another case, are
considered hearsay but admissible under FRE 804 (b) (1)

19
Downloaded From OutlineDepot.com

b. 804(b)(1) Testimony given as a witness at another hearing of the same or different proceeding or in a deposition
is NOT excluded by the hearsay rule if:
i. The party against whom the testimony is offered had an opportunity and similar motive to develop the
testimony by direct, cross, or redirect examination
c. Requirements for admissibility:
i. Identity of parties
1. The party against whom the testimony is being offered must have been a party to the earlier
proceedings, must have had an opportunity to examine the witness when the testimony was
recorded, and must have a similar motive as previously
ii. Identity of issues
1. Issues in the prior proceeding must be substantially the same
iii. Unavailability of the witness
1. Witness must be unavailable for a valid reason (death, physical disability, an inability or refusal to
testify)
d. (Travelers Insurance v. Wright) The testimony of a witness in a criminal case can be used in a subsequent
civil case if witness is not available but not against a person who was not a party in the prior criminal trial.
(Federal Rule)
i. Trying to recover on fire insurance where D hired someone to burn down the building with arson.
Testimony of arsoners at Ds criminal trial attempted to get into this civil trial. JC was not at prior criminal
trial so not admissible against him. This is a OK case
ii. In the federal rules, this testimony would not be allowed at JCs trial.
iii. Testimony from a prior criminal case can be introduced in a subsequent civil case when it appears
that:
1. Unavailable witness (804a) It is impossible to obtain testimony of witness who previously testified
2. Testimony at another proceeding (804b1) There was an opportunity to cross-x the witness by the
party against whom testimony is sought to be introduced
3. There is an identity of issues (must be about the same issue)
4. There is an identity of parties- opportunity and similar motive to develop the testimony by direct,
cross or redirect examination.
e. (U.S. v. Salerno) FRE 804 (b)(1) (former testimony) does not allow a criminal D to introduce GJ testimony
of a witness who asserts the 5th amendment privilege at trial
i. Mafia on trial for RICO. Rules of evidence do not apply at a grand jury. Witnesses give exculpatory
testimony at the grand jury.
ii. The motive of a prosecutor in questioning a witness before the grand jury in the investigatory stages of a
case is far different from the motive of a prosecutor in conducting a trial
iii. Even if the witnesses had testified differently in the grand jury testimony, it still could not be admitted b/c
there was no opportunity to cross-x etc. during the grand jury testimony
iv. Dissent: prevents the operation of the adversarial system of justice and hampers it- Stevens
II. Declarations Against Interest
a. When a hearsay declarant not a party to the action has made statements against her own apparent self-interest, such
statements are admitted since they are probably trustworthy
b. 804 (b)(3) Statement against interest
i. Statement at time of its making so far contrary to declarants pecuniary interest, or tended to subject
declarant to liability, or to render invalid a claim by declarant against another, that a reasonable man in
declarants position wouldnt make it unless believing it true
c. Requirements for admissibility
i. Unavailability
1. Declarant must be unavailable to testify at the time of trial
ii. Competency
1. Declarant must be competent, have personal knowledge of the facts stated, and not state these
facts in opinion or conclusionary form
iii. Declarations against the declarants interest
20
Downloaded From OutlineDepot.com

1. Declaration must be to declarants immediate prejudice when statement is made


2. Declarant must be aware that her statement is against her interest
3. The interest affected must be of a substantial nature and a pecuniary one
a. Some courts admit declarations that subject declarant to crim liability
d. Some courts have expanded the exception to admit declarations that subject the declarant to the risk of hatred,
ridicule, or social disgrace
e. (G.M. McKelvey v. Gen. Casualty) In civil cases, written confessions by unavailable 3rd parties can be
admitted
1. Employees who embezzled money and made signed statements admitting to it. The company
went after the insurance company for the money back.
2. A declaration against interest by one not a party or in privity w/ a party to an action is admissible
in evidence where:
a. The person making such declaration is either dead or unavailable as a witness due to
sickness, insanity or absence from the jurisdiction
b. The declarant had peculiar means of knowing the facts which he stated
c. The declaration was against his pecuniary (Insurance may sue the individuals to get the
money that was owed or they didnt have any money so how could there be a pecuniary
interest) or proprietary interest
d. He had no probable motive to falsify the facts stated
e. Probative value vs. risk of unnecessary risk of prejudice. Is this the only evidence of
these statements?
f. (U.S. v. Barrett) Statements against penal interest may be admissible if sufficiently corroborated
i. Buzzy, a prosecution witness, and Tilly, an alleged co-conspirator is dead before trial and Buzzy trestify
that Bucky and Tilly did it. Melvin, a defense witness, is going to testify that Bucky didnt do it and that
Tilly told Melvin that Tilly and Buzzy were the ones who did it (the stamp theft), not Bucky. A statement
against penal interest.
ii. Issue: What is the statement against interest?- Nothing inculpatory about Tillys statement. Because Tilly
can use this statement to impeach the witness because it would demonstrate that Buzzy was lying on the
stand.
iii. A declaration of interest by an out-of-court declarant is admissible if the remarks would tend to implicate
the declarant and have sufficient corroboration to show trustworthiness
g. Spent a lot of time on this case: (Williamson v. U.S.) Statements that are not self-inculpatory cannot be
used against others, even if they are made in a broader narrative that may be self-inculpatory
i. Williamson and Harris are drug smugglers. Harris made a deal and is the snitch in this case. Statements
made by Harris over a few days
ii. OConnor would remand the case for a more fact specific inquiry
iii. Kennedy and adds would allow in parts of the statement.
iv. FRE 804 (b)(3) does not allow admission of non-self-inculpatory statements, even if they are made within
the general manner
Declaration against Interest Rule 804(b)(3) Only come in under 804- need an unavailable declarant
i. Declarant must be unavailable
ii. At the time the person made the statement, the Statement must be
against pecuniary, possessory (proprietary) or penal interest
subjecting oneself to liability counts
1. Pecuniary money, I owe Bill $2
2. Possessory (Proprietary) property, This is your pen
3. Penal I robbed the bank
Rule does not include self respect humiliation, I sent the mean letter to Sue
iii. Must be so far contrary to interest that reasonable person would not
have made it if it were not true
1. Must have personal knowledge
2. Must know it is contrary to interest at the time made
21
Downloaded From OutlineDepot.com

iv. If exculpates the accused, corroborating circumstances must indicate


trustworthiness- not that it is accurate only that it is trustworthy
that the declaration is against his interest.
A statement that is self-inculpatory may still inculpate another and may be admissible
I. State of Mind
a. This exception allows the introduction in evidence of un-excited statements of present sense impressions indicative
of mental state made by out-of-court declarants
b. FRE 105 Limited Admissibility
i. When evidence which is admissible to one party or for one purpose but not admissible as to another party
or for another purpose is admitted, the court, upon request, shall restrict the evidence to its proper scope
and instruct the jury accordingly
c. FRE 803 (3) Then existing mental, emotional, or physical condition
i. A statement of the declarants then existing state of mind, emotion, sensation, or physical condition, but not
including a statement of memory or belief to prove the fact remembered or believed unless it relates to the
declarants will
d. Declarants state of mind is in issue
i. Whenever the declarants state of mind is in issue (i.e. her feelings, emotions, attitudes, knowledge, belief,
plans, intent, etc.) then the declarants out-of-court statements concerning her mental condition, if made at
the time such condition is in issue, are admissible
e. Declarants future conduct
i. Declarants extrajudicial statements concerning her mental state are admissible not only as proof of her
state of mind at the time the statements were made, but also to show that the declarant subsequently acted
in accord w/ her state of mind
f. (Adkins v. Brett) Evidence that goes beyond state of mind and tends to prove other material matters it is
not competent to prove is still admissible
i. Wife has a toy boy (Brett). Tort for alienation of affection and husband now sues Brett. Statement in
question was a conversation between husband and wife where wife admitted that she went automobile
riding and received affection from Brett and that husband was not.
ii. Correct jury instructions are important, could prejudice the case if incorrect
iii. Declarations of state of mind only exempted for showing state of mind of which the statements relate not to
prove what
g. Seminal Case- Important Case: (Mutual Life v. Hillmon) Evidence of an out-of-court declarants
intentions may be admitted to prove the declarant did what was intended
i. Hillmon and Walters leave west in search for gold. Hillmon doesnt come back from the trip he was shot.
Insurance company gets sued for the insurance money and argues that Hillmon is not dead. Insurance
company attempts to introduce into evidence letters from Walter to his family speaking of going to Witchita
with Hillmon. Walters is also missing.
ii. When a persons state of mind is itself a distinct and material fact in a chain of circumstances, it may be
proved by contemporaneous oral/written declarations
iii. An intent to do something is a state of mind and comes within the exception
iv. Circumstantial Guarantee of Trustworthiness: People intend the consequences that occur.
v. This exception is limited to the Declarants state of mind and no one elses state of mind or intention.
vi. The conduct itself is less probable than the statement of intention
1. Easier to prove conduct than state of mind, so need to use state of mind isnt great
h. (Shepard v. U.S.) Declarations pointing to past conduct are inadmissible
i. Medical Officer convicted of murder of his wife. Statements offered to prove the state of mind of wife say
that her husband was meant to kill her trying to prove she had a will to live. This was used to prove of an
act committed by someone else.
ii. Declarations of intention, casting light upon the future, have been sharply distinguished from declarations
of memory pointing backwards to the past
iii. The statement in Hillmon (high water line) looked forward, not backward as in the case here. If it was
allowed, then all recollections/memories would be admitted, destroying the hearsay rule
22
Downloaded From OutlineDepot.com

i. Skipped over in class: (U.S. v. Pheaster) State of mind hearsay may be admitted when it reveals the
declarants intent to do something with another person
i. The Hillmon doctrine states that when the performance of a particular act by a person is an issue in a case,
the intention to do the act may be shown. The jury may not infer from intention that the act was performed
j. (Zippo v. Rogers Imports) Surveys are generally allowed in, depending on the procedures used and how
they were conducted
i. Statements vs. Conclusions based on statements.
ii. Some cases hold that surveys are not hearsay at all b/c they are not introduced to show the truth of the
matter asserted, others hold they are hearsay but admissible b/c of the state of mind exception- what does a
person prefer? , but prefer as compared to what? What
iii. The test or inquiry of a survey should be
1. The need for the use of the survey at trial
2. The circumstantial guarantee of trustworthiness surrounding the survey
a. What is the value of the survey (the weight and credibility of the survey)
b. What does it mean?
c. Is it reliable?
d. If rendered bad it is a 403 issue and judge could not it to be substantially prejudicial.
State of Mind Rule 803(3)
i. Direct statements of what a person believes is hearsay if offered to prove the truth of the statement
ii. Out of court statements offered to prove declarants state of mind are treated as an exception to the hearsay rule
iii. Proponent must establish that the declarants state of mind is a fact of consequence
1. Declarants state of mind is not a fact of consequence in most lawsuits but can be in some
situations
a. Adkins SOM is relevant to show that wife was alienated
iv. State of mind can also be circumstantial evidence of future acts , but cannot be offered to prove past acts, with the
exception of will contests
1. Threats to kill can be offered to prove you killed
2. OK = I am going to Duluth tomorrow
3. NO = I went to Duluth yesterday, cannot be used to prove I went to Duluth yesterday
I. Medical Diagnosis or Treatment
a. Present physical condition
i. Physical condition must be in issue
1. If a persons condition at some time is in issue, the statements made at the time in issue as to his
condition are admissible (conduct also ok, i.e. limping)
2. To satisfy trustworthiness, some courts require spontaneity
ii. Statements to non-physicians
1. May be made to any competent witness, but must be trustworthy
2. The statement need not be made by the injured if the statement is for diagnosis or treatment (ex
mother explaining condition of a child)
3. The rule is not limited to statements of present symptoms, as long as it is made for diagnosis or
treatment (exmy back has been hurting for 6 months)
iii. Examining physicians
1. Examinations solely for the purpose of testifying at trial is now regarded to the same extent as a
treating physician according to 803 (4)
2. But only goes to diagnosis, not treatment
b. Past physical condition
i. General rule is that statements of past bodily condition are not admissible
ii. Minority rulestatements admissible when made to a doctor or other medical person to assist in diagnosis
or treatment (FRE 803.4)
c. FRE 803 (4) Statements for purposes of medical diagnosis or treatment
d. Statement for Diagnosis or Treatment Rule 803(4)- Disregard Tender Years Exception!!!
i. Statement for PURPOSE of medical diagnosis or tx
23
Downloaded From OutlineDepot.com

ii. Describing medical history, past or present symptoms, or


iii. Describing inception or general character of cause if
reasonable pertinent to the diagnosis or tx (Common law medical diagnosis and
treatment)
- The ID of the perpetrator is usually not pertinent. But in
child abuse cases, most courts treat the identity of the perp as reasonably pertinent to
treatment
iv. If the statements describe present symptoms they
may also fit as present sense impressions (803-1) or statements of present bodily
condition (803-3)
1. Still must prove relevancy
2. Federal Rules suggest that
statements made to an expert witness qualify if they are for the purpose of
diagnosis
a. May be motivated by litigation
prospects
3. Statements need not be made to the one
treating
a. Ex. Sick persons statement to the cab driver, Take me to the hospital,
I have stomach pain
Prior Statement of Identification Rule 801(d)(1)(C)- Excited Utterances IDing the defendant includes its own impeachment
characteristics- made without thinking
Prior identifying statements are treated as an exclusion from the definition of hearsay if:
1. The declarant had personal knowledge
2. The declarant testifies (declarant must be available and testify- either P or D can call the witness.),
and
3. is subject to cross examination about the statement
II. Prior Identification- Definition of unavailability under 804 does not apply to 801!- Owens
a. Extrajudicial identifications that may or may not be confirmable at trial, usually admissible b/c an identification at
the time of the event or act in question is more reliable than a later ID
b. However, if the circumstances of the identification make it suspect, then the prior ID is out
i. Exproperly conducted police lineups are admissible, but any prompting of the witness at the time of the
identification will make the ID inadmissible
c. Prior ID is admissible ONLY if the declarant is available at trial to be cross-examined or the prior ID comes within
the former testimony exception to the hearsay rule
d. FRE 801 (d)(1) Prior Statement by Witness (not hearsay if)
i. Declarant testifies at trial and is subject to cross concerning statement, which is:
1. Inconsistent w/ the declarants testimony and was given under oath; or
2. Consistent w/ declarants testimony and is offered to rebut an express or implied charge against the
declarant of recent fabrication or motive; or
3. One of identification of a person made after perceiving the person (no objects)
e. Generally a show-up or line-up or sometimes a photo arrangement
f. (U.S. v. Owens) Testimony is still admissible if witnesss inability to testify to certain aspects is b/c of
memory lossthe 6th only grants a confrontation of the witness
i. Prison guard gets beaten with a metal pipe. Photo shown of D to prison guard while in the hospital and
prison guard ids the D. At trial he couldnt remember any details of the hospital or any visitors. (The
available witness who is unavailable).
g. 6th Confrontation Clause gives D the right to confront the witnesses against him, but this guarantees only an
opportunity for effective cross-x, not cross-x that is effective in whatever way the defense might wish- this might
pose a 6th Amendment problem if there is no memory of anything.
III. Past Recollection Recorded- Allows a witness to testify to his or her own prior hearsay

24
Downloaded From OutlineDepot.com

a. A witness with no present memory of the facts observed but that made a record of those facts, the record can be a
hearsay exception if a proper foundation is first laid
b. Present recollection revived
i. Something shown to revive the memory of a witness while on the stand, standard practice, normally
allowed. Once refreshed they can testify to what they know.
ii. For practical reasons it is usually better to refresh a witness before they testify
c. FRE 612 Writing Used to Refresh Memory- Writing must be disclosed if reviewed by the witness.
i. If a witness uses a writing to refresh memory for the purpose of testifying (either before court or while
testifying) then an adverse party is entitled to have the writing produced and to cross-x the witness. None of
the materials used to do this has to be admissible into evidence.
ii. Witness must be specific as to what he or she remembers- e.g. whether she remembers writing down the
number on the piece of paper or remember seeing the license plate.
d. FRE 803 (5) Recorded Recollection- Not independently significant as a piece of physical evidence. The only
significant part of it is what was written down on the piece of paper, which can be read into the record but not
introduced as evidence.
i. A memo or record concerning something a witness once had knowledge of but now has lost recollection,
shown to have been made when the matter was fresh in the witnesss mind may be admitted, but may not
be received as an exhibit unless offered by the adverse party
a. Unless witness can prove that this was a present sense impression there still may be a
hearsay problem. Unless the police officer can get the witness to adopt what he wrote
down by initialing and dating what was written down.
e. Requirements for admissibility
i. The witness must identify the writing as one made herself or under her direction
ii. The writing must have been made at the time when the facts recorded were fresh
iii. The witness must have forgotten the facts recorded so she cannot testify from stand
iv. The writing must be authenticated as an accurate record by having the person who recorded the facts testify
that she did so accurately
v. Since the contents of the writing are in issue, the best evidence rule applies and the original writing must be
introduced.
f. (Baker v. State) Evidence that is hearsay may be allowed to refresh the memory of a key witness, but only
for that purpose
i.
ii. The use of hearsay writing is for purposes of jogging memory, the document is not going into evidence
iii. The witness states his newly found memory, and it is his present recollection that he asserts as true, not the
contents of the document
e. Past Recollection Recorded Rule 803(5)
i. Memorandum or Record
ii. Concerning a matter about which witness once had
knowledge
iii. Witness now has insufficient recollection to testify fully and accurately
iv. Made or adopted by witness when memory was fresh
v. Reflects knowledge accurately
vi. Not offered as an exhibit

25
Downloaded From OutlineDepot.com

Past Recollection Recorded vs. Past Recollection Refreshed


* The document, not the witness provides * Witness is providing testimony that
the evidence -> hearsay has been refreshed
* Thus, proponent must establish the * Not offering out of court statement
trustworthiness of the document so it is not hearsay
* Need not establish trustworthiness
of the document
* Can test the testimony through

cross-examination, oath and


demeanor
Records of Regularly Conducted Activity Rule 803(6)
i. Memo, report, record or data compilation in any form
ii. Acts, events, conditions, opinions or diagnosis
iii. Made at or near the time
iv. Kept in the course of regularly conducted business activity
v. Regular practice to make the record
vi. Unless the source of information or method or
circumstances of preparation indicate a lack of trustworthiness (ex. eye toward litigation)
1. Must lay foundation by testimony of custodian or other witness who is familiar
with how the report was compiled; or
a. Foundation may be supplied by a certificate in compliance with Rule
902(11-12) or other rule or law where custodian or qualified witness
certifies to the listed elements
b. If offering authentication by certificate, you must provide written
notice Rule 902
2. Person with Knowledge you need not produce the author of the report, or the
person with personal knowledge, BUT....
a. The report must be based on somebodys personal knowledge and they
must have a business duty to make the report
b. Example: dock worker marks good received, sends report to
accountant. Acct. notes goods as part of inventory and would be a
business record. His report is based on the PK of the dock worker
c. Doesnt qualify is a non-worker makes the statement must have
business duty
3. If a document qualifies but includes hearsay, the statement needs to be taken out,
or it must qualify by itself under another exception/exclusion

I. Business and Public Records


a. Generally deemed trustworthy b/c businesses rely on these records for business operations
b. 803 (6)-(10)
c. Requirements for admission
i. Entry in regular course of business
1. Business refers to virtually any type of organization, and the records must relate to the primary
activity of the business. Records kept in anticipation of possible litigation do not qualify
ii. Form of records
1. Must be original, or at least the first permanent entry. 803 (6) allows records in any form if made
in the regular course of business
26
Downloaded From OutlineDepot.com

iii. Contents
1. Under 803 (6) entries of acts, events, conditions, opinions, or diagnoses are admissible. Must
have been within the personal knowledge of the entrant
iv. Timeliness
1. Entry must have been made at or near the time of the transaction
v. Entrants availability
1. Under 803 (6), the entrant does have to be unavailable for it to come in
vi. Authentication
1. Records must be properly authenticated- You need the custodian of the business records to do this.
d. (Johnson v. Lutz) Business records are inadmissible if they contain statements of people who are not
engaged in the business and have no duty to make the report
i. Police report excluded at trial. Report includes statements of bystanders. The statements was not made as
a regularly conducted part of any business. Bystanders has no duty to anything in this instance.
ii. The report filed by the officer was a part of his duty, but the contents of his report contained remarks by
people who are under no duty to make such statements during the course of THEIR employment
iii. The actual report could come in but the statements of bystanders are redacted
iv. Not a report that the business world would rely on.
e. (U.S. v. Vigneau) Business records exception does not allow statements in records made by a person not a
part of the business if statements are offered for their truth
i. Vigneau wires money involved in a drug smuggling organization. Form filled out by clerk. Trying to
prove that D was the one that asked the clerk to create the form. The document becomes the evidence.
But, the document contains the statement of a person who has no duty to report accurately.
ii. No safeguards of regularity or business checks automatically assure the truth of a statement to the business
by a stranger to it
f. (U.S. v. Duncan) Unauthenticated medical records can be admitted into evidence if they are part of a
compilation of business records
i. Hearsay within hearsay- Hospital records within insurance records admissible under 805.
ii. P to Hosp. (801d7a) to ins. Co. to ct. - Must maintain hearsay exceptions throughout the chain of custody.
iii. Person whose job it is to keep the records it the one who will lay the foundation for introduction of the
evidence into the case.
iv. Must be in the business of making the sorts of records that are introduced
v. A witness who lays the foundation need not be the author of the record or be able to personally attest to its
accuracy. There is no requirement that the records be created by the business having custody of them
vi. The main requirement is trustworthiness; b/c hospitals and insurance companies rely on the records to
conduct business, they are trustworthy
g. (Williams v. Alexander) Information in hospital records that do not pertain to treatment or diagnosis is not
admissible
i. Hospital statement to doctor regarding specifics of accident not admissible. Information is not relevant to
the diagnosis of the injury that the doctor would use.
ii. Newspaper reporters notes not a business record. The business of the newspaper is to sell papers. A
reporter is a profession. A newspaper is not the business record of the newpaper company.
iii. The business of a hospital is to diagnose and treat. Thus, the only memos and records pertaining to this
should be considered as made in the ordinary course of business
iv. It is not in the hospitals business to record statements describing the cause of the accident in which Ps
injuries were sustained
h. (Hahnemann U. Hospital v. Dudnick) Computer generated business records do not have to satisfy special
evidentiary requirements to be admissible
i. Expert testimony is not needed to introduce records that computer generate. A witness is competent to lay
the foundation for systematically prepared computer records if the witness:
1. Can demonstrate that the computer record is what the proponent claims
2. Is sufficiently familiar with the record system used
3. Can establish that it was the regular practice of that business to make the record.
27
Downloaded From OutlineDepot.com

ii. Witness may lay foundation for systematically prepared cpu records as long as witness:
1. Can demonstrate that the computer record is what the proponent claims
2. Is sufficiently familiar w/ the record system used; and
3. Can establish that it was regular business practice to make the record
i. (Palmer v. Hoffman) Accident reports are not in the ordinary course of business, thus inadmissible under
the business records exception
i. Trains report of accident inadmissible hearsay. The railroad wants to introduce its own business records.
ii. Engineer had a duty to report actually. Justice Douglas says that the business of the train is not to keep
records of the accident report.
iii. Records of the business of the business activities
iv. The report was not made in the systematic conduct of business, its primary purpose is in litigationas
such, the reports trustworthiness is suspect
j. (Lewis v. Baker) Accident reports allowed if not solely prepared for litigation purposes and are pursuant to
regular procedure b/c level of trustworthiness is different
i. CF Powell- Train accident records admissible because the law requires that these sorts of records are
business records of the train company. And the person who made the record was not involved in the
accident and it was not made in the name os self interest.
ii. The mere fact that a record could ultimately be used in litigation does not mean that it must be
excluded. There must be a motive to fabricate on the part of the preparers before the business records will
be excluded.
iii. Distinguished Palmer v. Hoffman b/c the engineer that prepared the report in that case was personally
involved in the accident and knew at the time he was making it that there was a probable law suit to come
of it
1. Thus, the level of trustworthiness is different
k. (Yates v. Bair Transport) A doctors report, even though made in anticipation of litigation, can be
introduced into evidence if there are guarantees of its trustworthiness
i. Doctors reports prepared in preparation for litigation admissible as to the adverse party. Keeper of the
records must testify if proffering party is offering them.
ii. If the records dont plan on using or if they are not records that are prepared for the preoffering /proponent
partys litigation then they are not business records prepared in preparation of litigation
iii. Thus, the 3 Dr. reports requested by D and made by Ds choice of Dr.s are admissible if offered by P. The
Dr. reports prepared by Ps dr.s are not

Public Records Rule 803(8) (may overlap with 803(6))


i. Records, reports, statements or data compilations; any form
ii. Public offices or agencies
Navy example- it is a department of the govt. Ct of App. ruled
that a dept. fit under the definition of an agency.
iii. Setting forth:
A. Activities of the office/agency checks issued, licenses, court entries, etc.
B. Matters observed under public duty, except matters observed by police (ex.
Weather reports, minutes of meetings, inspectors reports)
C. Factual findings/evaluative reports in civil actions and when offered by the
accused in criminal actions
a. Factual findings are given a broad interpretation and include opinions
or conclusions of a factual nature such as the cause of an accident
author of report may not have personal knowledge
b. Legal conclusions are excluded
iv. Should be admitted unless the sources of information or other circumstances indicate a
lack or trustworthiness
1. In assessing trustworthiness, consider...
a. Timeliness of investigation
28
Downloaded From OutlineDepot.com

b. Investigators skill or experience


c. Whether a hearing was held (also the type of oath
taken, hearing, PK)
d. Possible bias view toward litigation
h. Many exceptions under 803(8) also fall under 803(6)
Public Records Exception
iv. Can be put into evidence very conveniently b/c are admissible w/o sponsoring witness.
v. Reasons public records are allowed
1. Govt officials are considered trustworthy
2. Legal duty to be accurate provides incentives to be careful
3. The govt employees tend to be more neutral, thus more genuine
vi. 803 (8) Public Records and Reports
l. (Beech Aircraft v. Rainey) A court may admit evidence an investigative report containing factually based
conclusions or opinions on the specific issue to be decided by the fact-finder
i. Evaluative reports admissible- regardless of whether they include facts or conclusions/opinions
ii. FRE 803 (8) provides that public records and reports are not excludable as hearsay. The definition of
public records and reports includes factual findings resulting from an investigation
iii. As long as a conclusion or opinion is based on a factual investigation and satisfies the trustworthiness
requirement, it is admissible w/ rest of the report
iv. Unless the sources of information or other circumstances indicate lack of trustworthiness.
v. Allows litigators to draw on govt studies, an independent basis for substantive evidence to get into court
m. (U.S. v. Oates) Evaluative and law enforcement reports are inadmissible against defendants in criminal
cases
i. Possession of heroin case. Chemist who tested the drugs doesnt show up for court.
ii. If a record qualifies as a public record to let it stand also as a business record would be redundant. If a
record in a criminal matter is not admissible as either a business record or a public record it wouldnt be
admissible as the other option. 803(6) AND 803(8) or some other rule to get the actual document into
evidence.
You may still be able to get the conclusion read into the record (e.g. past recollection
recorded) but you couldnt get the document itself into evidence.
iii. The reports of the chemist were written assertions offered to prove the truth of the matters asserted in them.
They do not fall within the business records exception b/c they are govt reports setting forth factual
findings
iv. The documents cannot qualify as public records and reports as they were prepared by law enforcement
personnel in connection w/ an investigation.
v. The business records exception must be viewed in light of the public records exception
n. Skipped Over in Class! (U.S. v. Grady) Police reports are admissible under public records if prepared
pursuant to a routine duty to report the facts
i. The public records exception contains a restriction on law enforcement personnel reports to prevent
prosecutors from proving their cases be putting into evidence officers reports of contemporaneous
observations of crime
ii. The difference b/w Oates is the subjective nature of the elements going into the reports
iii. Court based holding on policy underlying the FRE 803 (8)(b)
iv. Trustworthiness, motive b/c the observations were so routine and Congress did not intend the rules to
exclude this evidence
i. Miscellaneous Exceptions
i. Rule 803(7), (10) Absence of Notation
1. Use proof to show payment was not made
2. Look in records in D has a drivers license
ii. Rule 803(9) Vital Statistics, (11) Religious Records
1. Likely would not fall under business records

29
Downloaded From OutlineDepot.com

iii. Rule 803(17) Market reports, commercial publications


1. Use records that you made a phone call to Hamline. Introduce the phone book to
prove Hamlines phone number
2. Prove the price of a stock
iv. Rule 803(18) Learned Treatise
1. Impeach an expert witness or use it as substantive evidence to establish a point
you are making, without calling a witness
v. Rule 803(22) Judgment of Previous Convictions
1. Inconsistent statements/pleas can be relevant
2. Collateral Estoppel effect may exist
3. Exception - Declarant Must be Unavailable Rule 804
Must first establish that the declarants testimony is unavailable under Rule 804(a), which
includes....
1. Valid claim of privilege
2. Persistent refusal
3. Lack of memory of the subject matter
4. Death, mental or physical illness
5. Absent and outside the subpoena power (state border)
IV. Miscellaneous Exceptions: Judgment of Previous Conviction
a. Prior Criminal Convictions
i. Traditional viewa criminal judgment is not admissible in a subsequent civil proceeding
ii. Modern trendpermit convictions to be entered, although some jurisdictions limit admissibility to felony
convictions, FRE 802 (22)
iii. Rule applies to the judgment of conviction, not necessarily the testimony
iv. 803 (22)
1. Only applies to major convictions
2. A guilty plea can be independently admissible, especially in major crimes, maybe not in minor
crimes (traffic tickets, etc.)- 801d2a!!! Admission! 801d2b- adoption of what the govt said.
Transcript is the business record of the court reporter.
b. Prior Civil Convictions
i. NOT admissible in a subsequent criminal trial b/c the standards of proof are different
c. Prior Acquittal
i. The prior acquittal is excluded as hearsay in subsequent civil cases. Rationale is that someone was
acquitted under standard of proof beyond a reasonable doubt does not mean that they may not be liable on
another aspect
d. 803 (11) (12) (13) (14) (15) (16) (19) (20) (21)
e. (Stroud v. Cook) A misdemeanor conviction is admissible in a later civil action
i. A judicial document is hearsay by definition, but the FRE specifically exempts a judgment of a felony
conviction but not a misdemeanor conviction
ii. The motivation to defend at the misdemeanor level is often minimal at best
I. Rule 807- Residual Exception - NOT AN ANSWER ON THE EXAM- DO NOT USE THIS ON THE EXAM!!!
1. The statement must:
a. Have equivalent circumstantial guarantees of trustworthiness
b. Be evidence of a material fact
c. Be more probative on the point than other evidence
d. Be consistent with the general purposes of these rules (see Rule 102)
and the interests of justice
e. Be disclosed in pretrial notice (not always followed)
2. The issue frequently comes up whether this exception should be a grant to trial judges to continue to use
common law power to admit otherwise reliable hearsay, or should be used rarely only where this type of
evidence was not contemplated by the drafters of the rule.

30
Downloaded From OutlineDepot.com

a. Some courts say if the hearsay is close to an exception but nearly misses, it should not be allowed
in under Rule 807
i. Example: grand jury or preliminary hearing testimony
I. The Future of Hearsay
a. 804 (b)(5) provides that if the court determines that:
i. the statement is offered as evidence of a material fact,
ii. the statement is more probative on the point for which it is offered that any other evidence, and
iii. the purposes of these rules and the interests of justice will be best served by admission of statement, then
statement should be admitted
b. (Turbyfill v. Intl Harvester) The catch-all exception can be met through circumstantial guarantees of
trustworthiness
i. Dealing w/ hearsay through the discretion of judges poses problems
1. Judicial competence, some are elected, etc. Policing this would be problematic, especially on the
state court level
2. Vesting this authority in judges would damage predictability, and would result in fewer settlements
c. (U.S. v. Dent) GJ testimony does not come within one of the specific hearsay exceptions, but may be
admissible via the residual exception
i. GJ testimony may not be admitted during a criminal trial under the catch-all exception of FRE 840 (b)
(5)
d. 807 Residual Exception
i. A statement not covered by 803-804 but has equivalent circumstantial guarantees of trustworthiness, if
the court determines that:
1. The statement is offered as evidence of a material fact
2. The statement is more probative on the point for which it is offered than any other evidence which
the proponent can procure through reasonable efforts
3. The general purposes of these rules and the interests of justice will best be served by admission of
the statement into evidence.
f. The Confrontation Clause
i. Has to do only with a CRIMINAL defendant, and he/she can object on grounds that constitutional rights
have been violated.
ii. Provides that the accused shall be able to confront their accuser
g. (Ohio v. Roberts) An unavailable witnesss out-of-court statement can be admitted against the accused if
the statement had an adequate indicia of reliability
i. Preliminary hearing is a hearing regarding probable cause to determine whether the case can go forward.
Previous cross-examined prior trial testimony is admissible. Forged checks charge and girlfriends
testimony (Ds witness) not what D thought it was going to be. Girlfriend is not available at trial to testify.
804b3
ii. 6th Amendment- Fair Trail right- right to be confronted against the witnesses against you, the right to an
attorney and so on
iii. If read literally the Clause would require any statement made by an out-of-court defendant be excluded.
This interpretation was not intended.
iv. ROBERTS TEST: Confrontation Clause restricts range of admissible hearsay in two ways
1. Prosecution must either produce the declarant whose statement is offered, or demonstrate that the
declarant is unavailable
2. Hearsay may be admitted only when the declarant is unavailable and then only when the hearsay
has the requisite degree of trustworthiness- Indicia of Reliability required!
3. When a hearsay declarant is not present for cross-examination at trial, the Confrontation Clause
normally requires a showing that his is unavailable. Even then, his statement is admissible only if
it bears adequate indicia of reliability. Reliability can be inferred without more in a case where
the evidence falls within a firmly rooted hearsay exception. In other cases, the evidence must be
excluded, at least absent a showing of particularized guarantees of trustworthiness.

31
Downloaded From OutlineDepot.com

b. Cases Interpreting Roberts:


i. Bourjaily v. US.- As long as 801d2e is satisfied nothing further need to be shown to surmount a
confrontation clause challenge. Treated Roberts as applicable to all hearsay admitted against a criminal
defendant, no independent inquire into reliability as long as the out of court statement fell within a firmly
rooted exception.
ii. Idaho v. Wright- admission of the doctor re child statements violated the confrontation clause. Reliability
could not be inferred from corroborating evidence. Only evidence corroborating is the circumstances
making of the statement
iii. White v. Illinios- Unavailability is a necessary step only when statements were mad in prior judicial
proceedings-
1. Justice Thomas concurring opinion- confrontation clause is implicated only insofar as they are
contained in formalized testimonial materials, such as affidavits, depositions, prior testimonial, or
confessions.
iv. Lilly v. VA- Statements against penal interest violated confrontation clause- Theory used by court was by
plurality. No majority theory put forward.
c. (Crawford v. Washington) Reverses Roberts (rule of law reversed, result in case would still be the same),
testimonial statements of unavailable witnesses allowed only if D had a prior opportunity to cross-examine
i. Husband and Wife approach victim re an alleged rape of Ds wife and D kills the V. There is slight
variation in stories regarding whether V pulled or grabbed at something while being attacked. Police
interrogated husband and wife separately giving them Miranda warnings. She is not testifying due to
spousal testimonial privilege, but didnt keep out of evidence any prior statements that met a hearsay
exception- statement against penal interest.
1. Interrogation as used in Crawford is not the same definition used in Crim Pro!- Rhode
Island v. Innis!!!
2. This is a new category of hearsay! If we are dealing with testimonial hearsay there is a 6 th
Amendment Confrontation Issue!
3. Doctrine of Interlocking Confessions- if the confessions of Co-Ds interlock then there is no
Confrontation Clause issues because the confessions corroborated each other- this case turns this
doctrine upside down slightly
ii. Who are witnesses?
1. Anyone who bears testimony?- Someone who has given testimonyAt any time
2. Is there a difference between a witness in front of a grand jury and that same person in the ER
making the same sort of statement?- In one instance the person is testifying in the other instance
the statement is made for the purposes of medical diagnosis.
iii. The Confrontation Clause commands not simply that evidence be reliable, but that reliability be assessed be
testing via cross-examination
iv. The Court reaffirmed the importance of the confrontation right and introduced a distinction b/w testimonial
and non-testimonial statements
1. CRAWFORD TEST: Testimonial, out-of-court statements offered against the accused to
establish the truth of the matter asserted may only be admitted where the declarant is
unavailable AND where the declarant has had a prior opportunity to cross-x
a. Grand jury testimony will always to testimonial and will never be admissible under this
test.
b. Dying declarations may be an exception to the 6th Amendment right to Ds right to
confront his or her witnesses against him, even if testimonial- Waiting for Giles v. CA to
come down from Sup. Ct.
c. A court could potentially still use the ROBERTS TEST for nontestimonial hearsay???
d. State courts could also apply Crawford more broadly if it chooses. Sup. Ct. only said
2. Testimonial Statements listed in Crawford on pg. 330:
a. Ex-parte in-court testimony or its funcational equivalent

32
Downloaded From OutlineDepot.com

b. Extrajudicial statements contained in formalized testimonial materials, such as


affidavits, despositions, prior testimony, or confessions.
c. Statements that ere made under circumstances which would lead an objective
witness reasonably to believe that the statement would be available for use at a later
trial.
3. Court did not fully define testimonial, but stated that the term applies at minimum to prior
testimony at a prelim hearing, before a GJ, at a former trial, and to police interrogations.
v. The Confrontation Clause does NOT bar the use of testimonial statements for purposes other than
establishing the truth of the matter asserted
vi. Discussion
1. No longer is reliability (alone) a way to admit evidence
2. To the extent that something is not a police interrogation, which would probably come in as
long as the victim is seen as calling for help rather than trying to aid the prosecution.
3. All that is necessary is an opportunity to cross, or the equivalent of an opportunity to cross-x
vii. Giles v. CA- The forfeiture by wrongdoing doctrine means now what
Not only does the D have to intend to make the witness unavailable to prevent testimony from
happening, question that is still unresolved is whether it has to be a witness.
II. Right to Confrontation Sixth Amendment
A. In all criminal prosecutions, the accused shall enjoy the right to ... be confronted with the witness
against him and to have compulsory process for obtaining witnesses in his favor Sixth
Amendment
1. Olden v. Kentucky 6th Amendment allows accused to cross-examine about matters
relating to bias, which is otherwise inadmissible
2. Chambers v. Mississippi Chambers at a race riot in MS kills a police officer and is
convicted. Accused has a right to ask about statements that might exculpate the accused
but were precluded by the hearsay rule. Other suspects testimony, Ds re-direct to impeach
his own witness and other witnesses should have been allowed- Harmful error- NOT a FAIR trail. This is
a 14th Amendment case. Witness Voucher Rule- Attorney vouches to the witness that he or she calls to
the stand.
3. United States v. Owens we saw that the right to confrontation was satisfied if the
declarant was on the witness stand and willing to answer questions
B. History
1. U.S. v. Mattox (1895) Supreme Court allowed a dying declaration and in a retrial,
former testimony against accused
2. Pointer v. Texas (1965) begins modern era when the Court made the right to
confrontation binding on the states through the 14th Amendment
3. Barber v. Page (1968) excluded preliminary hearing testimony of a declarant who was
in prison in a different state
a. Court required a good faith effort to produce the witness
4. California v. Green (1970) Death penalty case- 14th Amend. Fair trial case. The 6th
Amend. Cannot be applied to the extent that is denies D a fair trial. Admitted preliminary
hearing testimony where the witness appeared at trial and claimed lack of memory-
a. It seemed that the Court was moving to a position that to satisfy the 6th
Amendment, the State must produce an available witness
5. Dutton v. Evans (1970) the Court allowed a co-conspirator statement without requiring
that the state produce the declarant, because the evidence was not crucial or devastating
and was trustworthy
6. Mancusi v Stubbs (1972) the Court allowed former testimony from a witness who as in
Europe which in fact was crucial and devastating
C. Ohio v. Roberts was supposed to reconcile past inconsistencies
1. Preliminary hearing testimony by an unavailable declarant, called at the preliminary
hearing by the accused is admitted
33
Downloaded From OutlineDepot.com

2. Roberts Tests
a. In the usual case, the prosecutor must produce or demonstrate unavailability of
the declarant
b. If unavailable, the hearsay can be admitted if demonstrated indicia of reliability
c. Need not demonstrate reliability if it fits a firmly rooted hearsay exception
3. Crawford v. Washington Court overrules the Roberts test. The reliability test is
unpredictable and inconsistent with 6th Amendment purposes
a. Two inferences about the 6th Amendment
i. Aimed at the use of ex parte examination as evidence
against the accused
ii. Not limited to witnesses who take the witness stand
applied to hearsay declarants
D. Testimonial Statements
1. Testimonial statements against the accused made by declarant not testifying violates the
6th Amendment
2. If testimonial, it must be subject to cross-examination and the declarant unavailable, or it
cannot be admitted
3. Testimonial statements are not well defined but include.
a. Affidavits
b. Testimony from former trials
c. Preliminary hearings or grand jury testimony
d. Statements to police in investigation
e. Depositions, confessions; OR
f. Statements that declarant would reasonable expect to be used
prosecutorily
4. Statements that are NOT testimonial include
a. Business records
b. Co-conspirator statements in furtherance
E. (Davis v. Washington)- The Confrontation Clause of the Sixth Amendment, as interpreted in
Crawford v. Washington, does not apply to "non-testimonial" statements not intended to be
preserved as evidence at trial.
i. Statements are non-testimonial when made in the course of police interrogation under circumstances
objectively indicating that the primary purpose of the interrogation is to enable police assistance to meet an
ongoing emergency
ii. Statements are testimonial when (2-part test)
a. The circumstances objectively indicate that there is no such ongoing emergency, AND
b. That the primary purpose of the interrogation is to establish or prove past events
potentially relevant to later criminal prosecution.
iii. Accusatory statements made in the preliminary portion of a 911 call will be considered non-testimonial
iv. Davis primarily focuses on intent of the interrogator. Was he attempting to gather info likely to be
useful at trial or was he trying to meet an ongoing emergency?
Davis v. Washington (consolidated with Hammon v. Indiana)- 911 emergency call v. police
interrogation
Facts of the Case
Davis was arrested after Michelle McCottry called 911 and told the operator that he had beaten her with his
fists and then left. At trial, McCottry did not testify, but the 911 call was offered as evidence of the
connection between Davis and McCottry's injuries. Davis objected, arguing that presenting the recording
without giving him the opportunity to cross-examine McCottry violated his Sixth Amendment right to
confront his accuser as interpreted by the U.S. Supreme Court in Crawford v. Washington. The Washington
Supreme Court disagreed, finding that the call was not "testimonial" and was therefore different from the
statements at issue in Crawford.
Question
34
Downloaded From OutlineDepot.com

Under the U.S. Supreme Court's interpretation of the Sixth Amendment in Crawford v. Washington, may
statements made to police during investigation of a crime, though not made with the intent to preserve
evidence, be admitted in court without allowing defendants to cross-examine the person who made the
original statements?
Conclusion
Yes. In a 9-0 decision authored by Justice Antonin Scalia, the Court ruled that the Confrontation Clause of
the Sixth Amendment, as interpreted in Crawford v. Washington, does not apply to "non-testimonial"
statements not intended to be preserved as evidence at trial. Although McCottry identified her attacker to
the 911 operator, she provided the information intending to help the police resolve an "ongoing
emergency," not to testify to a past crime. The Court reasoned that under the circumstances, McCottry was
not acting as a "witness," and the 911 transcript was not "testimony." Therefore, the Sixth Amendment did
not require her to appear at trial and be cross-examined. Justice Clarence Thomas wrote a separate opinion
concurring in part and dissenting in part. He argued that though McCottry's statements were not
testimonial, the Court should not "guess" at the primary motive behind the statements. This case was
decided with Hammon v. Indiana.
F. Caufield v. State (MN Case)- MN Stat.- Lab report is admissible unless the chemist is subpoenaed 2 weeks
before trial.
1. 6th Amendment not offended by this statute b/c of Bourjaily and then Crawford comes out. Chemist was
not cross-examined and the Chemist is available. Testimonial hearsay an available, but not in court witness. By
not subpoening the chemist the court said that D waived the Constitutional right in the 6th Amend to confront
witnesses against D. Waiving a Constitutional right has to be knowingly, intelligently and voluntarily. No
waiver here.
2. No forfeiture here.
G. Forfeiture by Wrongdoing-
1. FER- 804- Intent is a part of this rule
2. State Court (MN in Her)- eliminated the intent element.
3. Courts eliminated the intent element of the wrongdoing (fraud) necessary for the admission of the
testimonial hearsay of the murdered victim.
4. Her and Giles created a murder forfeiture rule- If you are on trial for murder and V is dead and the
testimony of V comes into evidence 104a- preponderance
5. If indicted for murder there is pc (by preponderance of the evidence- judge wont overturn this b/c even
though lower standard the standards are similar)
H. Character Evidence
B. Michelson v. United States - ...this law is archaic, paradoxical and full of compromises and compensations by
which an irrational advantage to one side is offset by a poorly reasoned, counterprivilege to the other.
C. Three Questions You Need to Know
1. What is character evidence?
a. Character is a generalized description of a persons disposition, or of the disposition in respect to
a general trait
2. When can it be offered?
a. Rule 404 - not admissible to prove conduct in conformity with character; Propensity Rule
b. Exceptions
i. Character of the Accused Criminal- accused must choose to introduce character
witnesses
ii. Character of the Victim Criminal (usually self-defense cases) if put in issue by D,
Govt can put in evidence that it was the D with that type of character not the victim.
iii. Character of witnesses for truth and veracity (impeachment of witness) - attach on the
reliability of the witness
iv. Rule 404(b) - Fine Distinctions; other crimes
c. Rule 105 - Limited Admissibility - the same evidence is not always available for different
situations; jurors will get a limited instruction directing their attention to proper inferences from
which to take
35
Downloaded From OutlineDepot.com

3. 405- How can it be established?- opinion, reputation or specific instances of conduct?


a. Call a person about the target persons character
b. Rule 405(a) Reputation or Opinion
i. Are you familiar with Xs reputation at work for honesty?
ii. Do you have an opinion as to Xs character for honesty? Specific acts from which the
opinion is formed is NOT allowed in! When D offers reputation or opinion character
witness, Govt can ask about specific instances cant prove them out those. Govt can
only ask how the reputation or opinion might change if person had knowledge of
particular instances (examining the character evidence, probing into specific
instances/knowledge)
c. Rule 405(b) Specific Instances-
i. Character must be an essential element of a charge, claim or defense to use this!!!
may inquire as to specific instances that the witness heard (reputation) or knows (opinion)
ii. Must demonstrate your good faith to the court
d. Rebuttal - same rules apply
C. Rule 404(a) Evidence of a character trait is not admissible to prove action in
conformity EXCEPT
1. Character of the Accused (Criminal) if defendant chooses to introduce it
2. Character of the Victim (Criminal)
3. Character of the Witness prove truth and veracity; impeachment
4. Character in Sex Offenses, sexual assaults and child molestation cases
D. Prior Bad Acts: Rule 404(b) Fine Distinctions- Watch out for 403 prejudice as well!
1. To complete the story of the crime on trial by placing it in the context of
nearby and nearly contemporaneous happenings.
2. To prove the existence of a larger plan, scheme, or conspiracy, of which
the crime on trial is a part.
4. To prove other crimes by the accused so nearly identical in method as to earmark them as the
handiwork of the accused.
4. To show a passion or propensity for unusual and abnormal sexual relations
5. To show, by similar acts or incidents, that the act in question was no performed inadvertently,
accidentally, involuntarily, or without guilty knowledge.
6. To establish motive.
7. To establish opportunity, in the sense of access to or presence at the scene of the crime or in the sense of
possessing distinctive or unusual skills or abilities employed in the commission of the crime charged.
8. To show, without considering motive, that defendant acted with malice, deliberation, or the requisite
specific intent.
9. To prove identity.
i. Two Part Test to determine admissibility
1. Offense evidence must be relevant to an issue other than the defendants character
2. Evidence must possess probative value that is not substantially outweighed by its undue
prejudice and must meet the other requirements of Rule 403.
E. Character traits are excluded because
1. 403- Jury Prejudice too much weight as evidence of guilt or might punish the defendant for uncharged
acts whether guilty or not
2. Waste of Time on collateral matters
I. Character, Habit, & Custom
D. If character is in issue, than character evidence may be offered, usually to show a persons disposition to act in a particular
way
E. 405 tells us how to do 404
F. Government can not prove you guilty by evidence of your character. D has to put character into issue for the Government to
bring in evidence of character.
G. Methods of proving character (Rule 405-b)
36
Downloaded From OutlineDepot.com

1. Personal Opinion
a. One or more persons may offer their personal opinion (based on observation) as to some persons character
for honesty or some other trait
2. Reputation
a. Testimony as to a persons general reputation in the community is allowed whether the witness actually
knows the person in question or not
3. Specific Acts
a. Evidence of specific acts of a person leading to inferences to his character
H. Why is character evidence sometimes excluded?
1. A jury may focus too much on the moral infirmities of the accused than of evidence
a. A jury may feel the accused has committed more crimes than he has
2. Could be a waste of time litigating collateral issues
I. (Cleghorn v. NY Central & Hudson River Railway) Evidence of past events CAN be introduced if it is not used to
show the likelihood that the event happened again, but only to show the likelihood that people knew about the past
events
1. Train switch man was negligent in not switching the tracks for a train and someone was injured because of it-
evidence that he was drunk and that he is an alcoholic
2. Involves the use of character narrowly, not to show ultimate fact, just as a propensity to act a certain way
J. When character/reputation are directly at issue, truth is a defense & specific acts can be allowed
K. At CL, when character was directly at issue, reputation evidence probably would not be allowed. Under FRE, reputation as
well as opinion seems to be allowed here
L. (Michelson v. U.S.) Character witness can be questioned about specific bad acts of the D to determine the extent of
their knowledge of D
1. Quintessential example of how to offer reputation evidence
2. Prosecution can bring in arrest on cross when character put in issue by D. Prosecution must have a good faith basis
to ask the question.
3. When a D puts his reputation at issue and calls character witnesses, the prosecution may question to determine the
extent of their knowledge of D.
4. This includes questions concerning specific bad acts, even if they are not related to the crime for which D is on trial
5. Prosecution cannot start by offering character witnesses; the D has to open the door to the use of character evidence
M. (U.S. v. Carrillo) Extrinsic act evidence under the identity exception (404b) must have a high degree of similarity
such to mark the offense as the work of the accused
1. Bad character not admissible w/o specific exceptons
2. The identity exception of 404 (b) has a limited scope, it does not allow admission of extrinsic acts that are merely
similar, but only those that have such a high degree of similarity as to mark offense as handiwork of accused
3. Extrinsic act evidence admitted if:
a. It is relevant to an issue other than the Ds character
b. It possesses probative value that is not substantially outweighed by its undue prejudice, and meets
the other requirements of FRE 403
N. (U.S. v. Beasley) For pattern evidence to be admissible, the pattern must show identity, intent, plan, absence of
mistake, etc., pattern itself not a reason to admit
1. Doctor getting extra prescriptions starts a drug trafficking business.
2. Make sure the judge puts in the record why he or sho is making the curfew.
3. Pattern= series of acts that collectively identify the offender. It is used to show identity or membership in a
conspiracy, thus, temporal proximity of the acts is important
O. (U.S. v. Cunningham) Evidence of previous bad acts may be admitted to establish a context (or motive) for other
evidence
1. Nurse steeling Morphine/Demerol and switching it . She is also on probation and had showed a positive UA.
Evidence of her prior drug use admissible to show motive is ok.
2. Under 404(b), prior conduct cannot be admitted to show a propensity to act in accordance w/ the character
indicated by that conduct
3. It is admissible as long as the probative effect is not outweighed by its prejudicial effect
37
Downloaded From OutlineDepot.com

4.Prior offenses can be admitted if they show a specific propensity for certain behavior that would identify the accused
to the relief of others
a. Exsexual fetishes, animal cruelty, indecent exposure etc.
P. (Tucker v. State) To allow evidence of a collateral offense, the prosecution must show that D committed that offense.
There must be some kind of connection.
1. Live in North Las Vegas wakes up and finds a murdered victim in his home shot between the eyes. It happens again
and the state prosecutes. The State attempts to get in evidence of the previous victim that showed up in his house.
Inadmissible there is nothing that connects the body of the first victim and D.
2. Before evidence of a collateral offense can be admitted, the prosecution must establish by plain, clear, and
convincing evidence (New standard is by a preponderance of the evidence!!!) that D committed it
3. Federal Rules changed everything. Before the rules the standard of proof was higher. Reference to clear and
convincing evidence has changed.
Q. (Huddleston v. U.S.) In admitting evidence of other crimes under 404(b), a court doesnt have to make a prelim
finding that other acts were proved by a preponderance
1. Established that the new standard is preponderance of the evidence, and
2. 404(b) prohibits admission of similar act evidence unless it bears upon a relevant issue in the case, such as motive,
opportunity, or knowledge
3. Standard is evidence sufficient to support a finding by the jury not a preponderance of the evidence
4. The key section here is 104(b), which provides that when the relevancy of the evidence depends on an underlying
condition of fact, the court shall admit it subject to the introduction of evidence to support the fulfillment of
condition
R. Difference b/w 104 (a) and (b) is that a jury can weigh credibility of witness
F. Rule 406 Habit; Routine Practice
1. Evidence of habit of a person or routine practice of an organization is admissible if relevant to prove that
the conduct of a person or organization on a particular occasion was in conformity with the habit or routine
practice
a. Habit is a regular practice of meeting a particular kind of situation with a certain type of conduct,
or a reflex behavior in a specific set of circumstances. (Perrin v. Anderson)
2. Habit can be proved by
a. testimony of specific instances
b. opinion of habit of x
I. Habit
a. (Perrin v. Anderson) D in a civil case can use evidence of Ps prior acts to show a habit
i. V killed by copsevidence of who the aggressor was is admissible as evidence of habit, but not admissible
as character of the first aggressor by specific instances of conduct. Character evidence in these
circumstances is ONLY can be done through opinion or reputation evidence. A pattern of conduct is
admissible to prove conduct!
ii. Character can NEVER be predictive of conduct!!!
Beasley is distinguishable because it didnt happen enough times to make it habit.
iii. Conduct that develops character can NEVER prove conduct
Because Norm goes to Cheers every day and has become an alcoholic because of it cannot prove
that he will go back to Cheers again.
iv. You can also have corporate habits!
v. Under 404(a), character evidence used to show that an individual acted in conformity w/ his character is
subject to strict limitations
vi. Exceptions to the 404 (a) ban on the use of character evidence permit criminal Ds to offer evidence of
their own character or of the victims character
1. Once a D takes this initial step, the prosecution may rebut by offering contrary character evidence
vii. 405 evidence is only allowed when character is in issue in the strict sense
1. Character is directly in issue in the strict sense when it is a material fact that under the
substantive law determines rights and liabilities of the parties

38
Downloaded From OutlineDepot.com

b. Park, Leonard, & Goldberg Distinction between character and habit


i. Character evidence is NOT admissible in civil cases as circumstantial evidence of conduct, and IS
admissible in criminal cases under prescribed conditions
1. However, evidence of habit is freely admissible
ii. Character = a generalized description of ones disposition, or of ones disposition in respect to a general
trait, such as honesty, temperance, or peacefulness
iii. Habit = more specific, it describes ones regular response to a repeated specific situation. The FRE contain
no requirement that habit or routine practice be corroborated by other evidence; it by itself is sufficient
1. Examples of Habit:
a. Dr.s regular practice of advising patients of risks
b. A bars regular practice of serving intoxicated persons, to support the inference that it did
so on a particular occasion
c. A drinkers habit of getting drunk at a particular social club nearly every weekend to
show intoxication at the time of a hit-and-run
d. A RRs practice of not blowing a whistle at a particular crossing
c. (Halloran v. VA Chemical) Evidence of a deliberate repetitive practice may be admitted to show that the
practice was followed on one particular occasion
i. One who has demonstrated consistent response under given circumstances is more likely to repeat that
response when circumstances arise again, evidence of habit is generally admissible to prove conformity on
specified occasions
II. Similar Happenings- (Just briefly went over in class!)
a. Other Contracts
i. When the same parties are involved in other K transactions, evidence of such other Ks will be received if
relevant to show that probable meaning that the parties gave to the disputed K
b. Previous Accidents and Injuries
i. Prior Accidents
1. Prior accidents to others resulting from a condition or activity conducted by the D may be shown
to prove the Ds negligence in the present case.
2. Such evidence may be used to prove that D had knowledge of the danger involved, or that a
reasonable person would have
3. To have the evidence admitted, the proponent must show:
a. Substantial similarity of existing conditions, and
b. Close proximity in time between the two accidents
ii. Proof of subsequent accidents
1. Courts do not permit a party to show subsequent accidents to prove that a condition or product for
which the D is responsible caused an earlier injury to P
2. BUT, courts will permit such proof if it is used as circumstantial evidence that a dangerous
condition existed at the time of the accident being litigated
iii. Absence of other accidents
1. Courts do not admit evidence that there were no similar accidents or occurrences in order to show
that the D did not have knowledge of the danger
c. Application in Negligence Action
i. (Simon v. Kennebunkport) In a negligence action, evidence of prior similar accidents may be
admitted to show the existence of a defect, notice or causation\
ii. If relevant, is it unfair under 403?
iii. Sidewalk negligence- Injury prior similar accidents admissible
iv. The absence of other accidents may also be relevant
V. Subsequent Precautions- The law wants to encourage settlement and keep people out of court.
A. Rule 407 Subsequent Remedial Measures
1. Evidence of subsequent remedial measures are not admissible to show (liability)...
a. negligence

39
Downloaded From OutlineDepot.com

b. culpable conduct
c. defect of product/products design
d. need for a warning or instruction
2. Evidence is admissible to show feasibility of precautionary measures, if controverted.
3. Policy (Tuer v. McDonald)
a. Subsequent conduct is not an admission
b. Social policy of encouraging people to take steps in furtherance of
added safety
4. Rule 407 is not a rule of privilege, it is a rule of relevancy
a. It is discoverable if it may be admitted and is not covered by privilege
B. Rule 408 Offers to Compromise
1. Policy
a. Offering to compromise is not an admission of guilt/think they are
guilty
b. Encourage compromise and protect those while compromising
C. Rule 409 Payment of Medical and Similar Expenses
1. Evidence of payment or offers/promise to pay medical, hospital or similar
expenses is not admissible to prove liability
2. Beware of other payments mixed in with medical payments
a. Example car repair in addition to hospital bills after car accident
D. Rule 410 Inadmissibility of Pleas, Plea Discussions & Related Statements
1. Pay attention to the form of plea
V. Subsequent Repairs or Precautions
a. Introduction
1. Following an injury to P, evidence that the D repaired some condition on his premises that was alleged to
be defective and the cause of the injuries is generally NOT admissible to prove negligence or other
culpable conduct in connection w/ the event
2. Subsequent repairs ARE admissible to impeach the Ds witnesses, to show ownership or control of the
premises, or to show that the D was trying to conceal or destroy evidence
ii. 407 Subsequent Remedial Measures
b. Subsequent Change in Medical Protocol
i. (Tuer v. McDonald) Evidence of a change in medical protocol may NOT be admitted to rebut a
physicians statement that the new protocol would have been unsafe at the time he made the decision to
follow the old protocol that was then in effect
1. There are 2 justifications for excluding evidence of subsequent remedial measures to prove culpability
a. The subsequent conduct is not actually an admission
b. Public policy encourages people to take remedial measures
2. Testimony of a subsequent change is not admissible under feasibility or impeachment
c. Compromise
i. Civil cases
1. In order to encourage parties to settle their disputes out of court, courts will not permit a party to offer in
evidence the fact that the adverse party made an offer to compromise or settle
2. 408 Compromise and Offers to Compromise
3. Leading theory supporting exclusion is irrelevancesettlement offer not really probative b/c a party may
consider a settlement just to make peace or avoid litigation
a. However, a settlement greater than the cost of litigation could be probative
4. Supervening policy consideration
a. Partiality of the law toward out-of-court settlement, parties could be hesitant to offer or make
settlements if they think that might be used against them in court
5. BUT, if a party made an admission during compromise, that admission could be received in evidence
even if the compromise itself would not be admissible

40
Downloaded From OutlineDepot.com

ii. Non-Negotiation Communication from Plaintiff


1. (Davidson v. Prince) Evidence of a writing offering compromise is not admissible, but if a pre-
trial writing does not contain negotiations of compromise, it is admissible
a. Under 408, evidence of conduct or statements made in compromise negotiations is not
admissible, but this is not an offer to compromise
iii. Traffic offenses
1. (Ando v. Woodberry) A guilty plea in a traffic trial can be introduced in a subsequent civil trial is
admissible but weight of the evidence may be determined by Ds explanation
a. Despite that many Ds in traffic cases plead guilty as a matter of convenience, that should only
go to weight of evidence not to whether it should be admitted

CHAPTER SIX
Competency of Witnesses- General Rule: Every person is competent to be a witness. A person who is drunk, mentally ill,
adjudicated incompetent are all still competent to be a witness. Nor age requirement.
I. General rules to qualify as competent
a. Ability to communicate
b. Personal Knowledge- Rule 602- This doesnt apply to expert witnesses.
i. Witness must have personal knowledge regarding the matter testifying about
c. Obligation of truthfulness- Rule 603- must declare that you will testify truthfully by oath or affirmation.
d. Interpretors- Rule 604- oath or affirmation to make a true translation.
e. Competency of Judge as Witness- Rule 605- The judge may not testify at a trial he or she is presiding at.
f. Competency of Juror as Witness- Rule 606- may not unless question of whether extraneous prejudicial
information was improperly brought to the jurys attention on whether any outside influence was improperly brought
to bear upon any juror.
g. Who May Impeach- Rule 607- any party
h. Evidence of Character and Conduct of Witness- Rule 608
i. Impeachment by Evidence of Conviction of Crime- Rule 609
j. Religious Beliefs or Opinions- Rule 610- Evidence of beliefs or opinions of a witness on matters of religion is not
admissible for the purposes of showing that by reason of their nature the witness credibility is impaired or
enhanced.
k. Mode and Order of Interrogatories and Presentation- Rule 611
l. Time of competency
i. Refers to the condition of the witness at the time called to testify
I. Competency
A. Designed to protect the jury from unreliable evidence
1. A major issue at common law....
a. Religious belief
b. Convicted felon
c. Interested party
d. Dead Mans Statutes
e. Infancy
f. Mental Derangement
g. Connection with the tribunal judges, jurors
2. Remnants remain of many of these principles
a. Religious belief Oath Rule 603
b. Convicted felon Impeachment Rule 607
c. Interested party Impeachment Abel
d. Dead Mans Statutes abrogated in most states
e. Infancy
f. Mental Derangement
g. Connection with the tribunal judges, jurors Rule 605, 606

41
Downloaded From OutlineDepot.com

B. Infancy
1. Young children may testify as long as they have sufficient capacity to:
a. Understand the nature and obligation of the oath; and
b. Can perceive, recollect and narrate accurately
2. The issue is subject to the discretion of the trial judge
3. Even if the child is incompetent to testify, his hearsay statement may still be admitted
C. Mentally Impaired
1. Mentally impaired can be competent to testify if they have sufficient mental capacity to
perceive, recollect and narrate
D. Interpreters Rule 604
1. Interpreters are treated as expert witnesses
2. Some judges and lawyer confuse interpreters with court reporters
a. Interpreters must take an oath to interpret fairly and they must be qualified as an
expert
i. The ability to order from a menu in a foreign language does not qualify
you as an interpreter must take a test
E. Judges Rule 605
1. Sitting judges are incompetent to testify in the cases in which they preside
a. They can step down and become a witness; Cannot be both judge
and witness at the same trial
b. Same goes for attorneys
F. Jurors Rule 606
1. Juror may not be a witness and a juror in same case
2. In post-trial motions, juror may not testify about their thought processes or statements
made in deliberations impeach the verdict.
3. Jurors may testify about:
a. Extraneous prejudicial info (newspapers, unauthorized view)
b. Outside influence improperly brought to bear (bribes, threats)
4. Jurors may not testify that
a. Misunderstood instructions
b. Drunk or high
c. Chose to disobey instructions
d. Flipped a coin to decide the verdict
G. FRE have nearly eliminated competency as an issue at trial
1. Rule 601 makes everyone competent except as provided in the rules
a. Only exclusions...
i. Lack of personal knowledge Rule 602
ii. No oath or affirmation Rule 603
iii. Interpreters Rule 604
iv. Judges, jurors Rule 605, 606
2. If the issue is resolved by state law, the federal courts must apply state rules of
competency
a. If the prosecutor puts a 3 year old witness on the stand, a court may consider excluding the testimony under Rule
401-403 (trend to include)
I. Cross Examination
A. Rule 611(b) Scope of Cross Examination
1. Limited to subject matter of direct examination and matters affecting credibility of the witness
2. Credibility of a witness is a fact of consequence under Rule 401
3. Court has discretion to allow additional inquiry as if on direct examination
If you take the witness on direct, make sure not to waive your judgment of
acquittal rights.
B. Rule 611(a) Leading Questions
42
Downloaded From OutlineDepot.com

1. Leading questions are not used on direct examination except as necessary to develop the witness testimony
a. Hostile witness
b. Adverse party
c. Witness identified with adverse party- adverse witness
d. Preliminary matters
e. Refresh recollection
f. Witness with limited capacity age, mental capacity
g. Cross examination
C. Goals of Cross Examination
1. Discredit direct examination
2. Discredit other testimony
3. Corroborate favorable testimony
4. Produce favorable evidence for your own case
II. Impeachment
A. Credibility of a witness is a fact of consequence under Rule 401
1. All witnesses can be subject to an attack on their credibility
B. Ways to Impeach
1. Prior Inconsistent Statements
a. On the issue of credibility, you may introduce prior inconsistent statements made by the witness
b. Statements are offered on the limited issue of credibility
c. Statements are not offered to prove that the witness was telling the truth when the witness made
the out of court statement hearsay
d. Rule 612 - You first refresh the witness memory by asking Do you remember me asking....and
you said...
2. Contradictions
a. To develop contradictions you may ask on cross examination about collateral issues
i. Test of collaterallness: is could the fact, as to which the error is predicated, have been
shown in evidence for any other purpose independently of the contradiction. (State v.
Oswalt)
ii. Rule 608- When inquiring into a witness character for truthfulness you can use prior
conduct you cant prove out the conduct with extrinsic evidence.
ii. You may not disprove statements on collateral issues by extrinsic vidence
iii. Extrinsic evidence through the testimony of a different witness
iv. Intrinsic evidence through the testimony of the witness
b. If a document contradicts a witness testimony....
i. You can show witness the document and ask if it refreshes their recollection (intrinsic)
ii. You cant introduce the document if it requires authentication from another witness
(extrinsic)
iii. You maybe able to introduce the document if the witness you are contradicting will
authenticate the document
3. Defect in Capacity
5. Bad Character for Truth and Veracity
a. Opinion/Reputation
b. Bad Acts
i. Rule 608(b), policy rule Character for truthfulness. Can NOT offer specific instances to
prove Allows you to impeach through cross examination about bad acts that reflect
directly on the issue of credibility (ex. not reporting a conviction on an application)
Husband and Wife divorce and commits a crime. This could show bias
Impeachment of Witnesses
I. Contradiction
a. 2 basic methods of impeachment

43
Downloaded From OutlineDepot.com

i. Cross-x
ii. Introduction of extrinsic evidence to contradict something the witness has said
1. Attacks the witness w/ evidence of contrary facts
b. Generally, when extrinsic evidence is not allowed is b/c it involves collateral matters
c. (State v. Oswalt) A witness cannot be impeached upon matters collateral to the principal issues being tried
i. Impeachment w/ collateral matters is excluded in order to:
1. Avoid confusion of the issues, and
2. Prevent unfair advantage over a witness unprepared to answer question concerning matters
unrelated or remote to the issues at hand
ii. A test: could the fact, as to which error is predicated, have been shown in evidence for any purpose
independently of the contradiction?
d. Didnt go over in class- (U.S. v. Copelin) Testifying D can be impeached by own statements even if
impeachment evidence would be inadmissible collateral matter if obtained through another witness
i. Discussion: What facts are admissible
1. Facts are not collateral if they involve substantive issues
2. Facts are not collateral if they are independently provable apart from extrinsic evidence (questions
the answer to might show bias or the witnesss lack of first-hand knowledge)
3. Facts are not collateral if they are linchpin type facts, that as a matter of human experience one
could not be mistaken (that fact stated was impossible, etc.)
II. Character of the Witness
a. Whenever a witness takes the stand, he puts his character for honesty and veracity in issue (i.e. His Character for
Truthfulness). Thus, he can be impeached by evidence that his character is such that he would lie under oath
b. Prior bad acts
i. Most courts allow cross-x as to prior bad acts if they are clearly probative of veracity and do not involve
unreasonable risk of prejudice (Character for truthfulness)
ii. BUT extrinsic evidence cannot be used to prove past misconduct
c. 404
d. Impeaching party can offer:
i. Generalities about the accused showing that he has a character for truthfulness
ii. Subject to restriction, specific misconduct that did not result in conviction but did result in untruthfulness
(608)
iii. Prior criminal conviction (609)
e. Limitsgenerally only applicable when about the witnesss character
f. (U.S. v. Owens) Ds may be cross-examined about character for truthfulness, and if D lies about prior acts
of intentional falsehood then lawyer cannot independently prove that D lied.
i. D convicted for murder of wife in military court of appeals. She was shot as she was driving away and D
claims that he was inspecting the gun, meant to push the trigger but didnt think there was bullets in the
chamber. At trial he was asked about an application he filled out to become a warrant officer in the
military. On the form he left out a conviction on the form regarding a previous assault. But, govt couldnt
prove that he did leave out a prior conviction because proving the previous conviction does not go towards
proving the witness character for truthfulness
ii. Lawyer can only ask witness to answer questions that go towards character for truthfulness. Cant inquire
whether the witness is lying to questions
iii. Can the prosecutor prove up extrinsically if the D denied the claims by producing a document showing
the falsehood? No.
iv. Past convictions that have nothing to show a propensity to lie would be excluded (i.e. a prior conviction for
marijuana possession, etc.)
g. (U.S. v. Drake) Cribari DOES NOT agree with this case!!!- Cross-examination questions alone cant
constitute extrinsic evidence
i. Although the questions did refer to records not in evidence, cross-x questions alone cannot constitute
extrinsic evidence

44
Downloaded From OutlineDepot.com

ii. If you get the witness to admit that what he said earlier was wrong that is ok and a form of impeachment by
contradiction
iii. Ct. said this didnt violate 608(b) b/c there was no extrinsic evidence that was introduced, just cross-
examination
h. (U.S. v. Saada) 806 does not modify 608(b)s ban on the introduction of extrinsic evidence in the context
of a hearsay declarant
i. 806 allows impeachment of a hearsay declarant only to the extent that impeachment would have been
permissible had the declarant testified himself
ii. Thus, the plain language of the Rules prohibits the use of extrinsic evidence to impeach a hearsay declarant.
c. Convictions- Rule 609
i. Rule 609 Cant inquire into the conduct of the crime behind the
conviction!!!divides convictions into 3 groups
1. Dishonesty or false statements 609(a)(2)
2. Crimes punishable by one year +
(usually only felonies grade crimes, shall be admitted for witness who is not the
accused (may also include common law misdemeanors), but must use balancing
test to determine whether admissible if witness is D 609(a)(1)
3. Misdemeanors not involving
dishonesty/false statement are not admissible, unless punishable by more than a
year (common law misdemeanors).
i. Although the rule says dishonesty or false statements, courts require
that the conviction involve false statements (perjury, criminal fraud
embezzlement or false pretense, lying on a federal form (loan
application, passport, ect.))
ii. 803 (22)- ??? Prior Conviction document of someone other than the accused not
excluded by hearsay when relevant to character for truthfulness of witness other than the
accused.
ii. If witness is the accused, the court may admit the conviction if the probative value (on
the issue of credibility) outweighs its prejudicial effect to the accused. If witness is not
the accused then there is no 403 inquiry.
1. Different from Rule 403 in a close case the court should exclude the evidence
2. In balancing probative value and the prejudice to the accused, the court should
consider...
i. Impeachment value of prior crime
ii. Point in time of conviction and the witness subsequent history
iii. Similarity between past crime and the charged crime
iv. Importance of the defendants testimony
v. Centrality of the credibility issue
iii. If the conviction is offered under Rule 609, it must relate to credibility
iii. If the conviction is older than 10 years
i. Prior Convictions
i. 609 Allows impeachment for any crime involving dishonesty or false statement, regardless of whether
a felony or misdemeanor
j. (U.S. v. Sanders) When D is a witness, Evidence of prior convictions not involving dishonesty are not
allowed for purposes of impeachment
i. D on trial for a drug charge has prior conviction of a previous drug charge entered
ii. Evidence of similar offenses can be highly prejudicial. It may do little to impeach his credibility but it can
obviously prejudice him
k. (U.S. v. Wong) Federal courts cannot apply the balancing test involving dishonesty or false statement for
impeaching a D, it must admit the evidence
i. 609 has not 403 inquiry. If it fits it gets in!!!

45
Downloaded From OutlineDepot.com

ii. 609 (a) distinguishes b/w convictions for crimes punishable by imprisonment of more than one year and
convictions for crimes involving dishonesty or false statement
iii. The former are admissible only after the court applies a balancing test
b. (U.S. v. Brackeen) Robbery is not a type of dishonesty to satisfy 609 (a)(2)
i. The crime must involve conduct of dishonesty or false statement (conduct of dishonesty or false statement
must be an element of the crime- THIS LEAD to the 609 AMENDMENT)
i. Dishonesty CAN include stealing, but its narrow meaning requires deceitful behavior or a disposition to lie,
cheat, or defraud.
AMENDED 609(a)(2)- (2) evidence that any witness has been convicted of
a crime shall be admitted if it involved dishonesty or false
statement regardless of the punishment, if it readily can be
determined that establishing the elements of the crime
required proof or admission of an act of dishonesty or false
statement by the witness.
c. 609b- Time Limit- 10 years or release of the witness from the confinement unless probative value is
substantially outweighed by the risk of prejudice.
d. (Green v. Bock Laundry) Balancing test applies only to criminal defendants
i. Congress intended the balancing test to apply ONLY to criminal defendants.
ii. 609 (a)(1) mandated reception of convictions in civil cases
e. (Luce v. U.S.) D must testify to preserve claim of error on admission of prior conviction
i. Prior conviction for rape. To raise on appeal the wrongful impeachment with a prior conviction when you
choose not to testify you waive your right to preserve the error because you didnt suffer the harm. You
must testify, you must suffer the harm.
ii. The appellate court must have a complete record in order to weigh the probative value of a prior conviction
against the prejudicial effect
II. Prior Statements to Impeach or Rehabilitate
a. 613 (a) Examining witness concerning prior statement
b. 801 (d)(1)(B)
c. A prior inconsistent statement may not be proven w/ extrinsic evidence when the statement is being offered
solely for collateral impeachment
d. (Coles v. Harsch) A mere general question on cross-x is insufficient to lay a foundation to impeach a
witness for prior inconsistent statements
i. The purpose for laying a foundation is to afford all witnesses opportunity to recall a fact before they may be
assailed as dishonest
ii. Thus, a witness can be impeached by prior inconsistent statements, but before this can be done, the
situation must be related to him, and if he has made such statements he should be allowed to explain them
iii. Every witness whose testimony is shown in conflict w/ a previous statement made by him is not necessarily
a dishonest person
iv. This rule of evidence is intended not only the dishonest witness, but also to all the witness ample
opportunity to recall a fact before assailed as dishonest
e. (Tome v. U.S.) Prior consistent statements are non-hearsay if offered to rebut a charge of
fabrication/motive and can then be used as substantive evidence 801(d)(1)(b)
i. A prior consistent statement is in the same category as a declarants inconsistent statement made under oath
in another proceeding; it can be used as substantive evidence.
ii. Non-hearsay status is granted to all prior consistent statements, but they must meet the requirement of
being offered to rebut an alleged motive
iii. A prior consistent statement made prior to the time when the motive to lie arose has no relevance to rebut
the charge that the in-court testimony was the product of motive to lie
III. Psychiatric Condition
a. Some courts allow a witness to be impeached by testimony from a treating physician as to the psychiatric condition
of the witness being impeached

46
Downloaded From OutlineDepot.com

b. Should be viewed as defects in the mental capacity of a witness whether mental, sensory, etc.
c. Memory and Intelligence
i. Generally can be proved by cross-x and extrinsic evidence (demonstrating the problem to lay the
foundation and then argue that character aside this person does not remember accurately and to not let
lawyer pry the memory and ability to remember it would be extremely prejudicial.
ii. Comments to 601
d. Psychiatric
i. Under the influence of alcohol or drugs can be shown by cross or extrinsic
ii. Chronic use/addiction is generally not allowed in, cannot be shown extrinsically
1. Still under case-by-case basis, fed. ct.s generally dont allow it, state ct. might
e. (U.S. v. Lindstrom) Evidence of a witnesss mental disorders that may affect her motives for testifying are
admissible to impeach the witnesss credibility
i. Several types of emotional or mental disorders may materially affect the accuracy of testimony, thus are
probative to credibility
1. Exa schizophrenic may have difficulty distinguishing fact from fantasy and may have memory
distortion from delusions
2. The issue of a witnesss credibility must be determined by the jury, the 6th gives a D the right to
confront
6. Rehabilitation
a. Once your witness is impeached you may attempt to rehabilitate by
i. Contradiction and reassertion subject to Rule 403
ii. Evidence of good character for truth/veracity Rule 608a
iii. Prior Consistent Statements Rule 801(d)(1)(B)
1. Rule permits the introduction of a declarants consistent out of court statement
of recent fabrication or improper influence or motive only when those
statements were made before the charged recent fabrication or improper
influence or motive
7. Bias
a. Although it is not specified in the FRE, extrinsic evidence may be introduced to show bias
i. Bias is a collateral matter
b. U.S. v. Abel Because membership to a common group is probative on the possibility of bias,
evidence is admissible even without a showing of adoption of the groups tenants
i. Gets you around Rule 404(a) - character
I. Bias
a. A witness that has some interest in the outcome of the trial or is otherwise biased or hostile
b. Examples of bias
i. Compensation for testimony
1. Might show the witness attempting to earn something for a certain testimony
2. exit is proper to ask a prosecution witness on cross-x whether there are any charges pending
against him, and whether he has been promise leniency
ii. Bias of friends and family
1. Relativesit may always be shown that the witness is related to one of the parties b/c this at least
infers a bias in that partys favor
a. However, evidence as to the bias, prejudice, or interest of the witnesss relatives is
generally NOT admissible (cant choose your relatives)
2. Friends
a. A person can choose their friends, so evidence of their bias or interest is generally
admissible for impeachment purposes
iii. Hostile statements
1. Animosity toward a party may be shown by adverse statements, or by the fact that the witness has
some dispute or litigation pending against the party
c. (U.S. v. Abel) Common membership in an organization is evidence of bias
47
Downloaded From OutlineDepot.com

i. Jail brotherhood swearing to lie for each other - Ds argument is 608b character for truthfulness cannot be
introduced of prior acts of extrinsic evidence. Govt argument is that the evidence is offered to show bias
(he has a reason to lie).
ii. Evidence that the bias person is a liar- let it in with limiting instruction to jury stating that the evidence
could go to the issue of bias, but not to determine whether or not the witness is telling the truth.
iii. 403 inquiry- probative value v. prejudice
iv. A showing of bias on the part of the witness has the tendency to make his testimony less probable than it
would w/o such testimony; thus the bias is relevant
v. The inference of bias varies depending on the type of organization involved, i.e., a book club would show
less bias that membership in a prison gan
Privileges
Protecting Relationships
A. Wigmores formulation of the conditions necessary to justify privilege...
1. Communication that originates in confidence
2. Confidentiality essential to the relationship
3. Relationship valued by the community
4. Injury to the relationship greater than benefit from accurate decision
B. Rationale
1. An alternative justification is sometimes advanced that privileges reflect privacy concerns that there are
enclaves of private communication that the government cannot extract from its citizens in order to have a
correct decision at trial.
2. Rationale is that the attorney is the professional and is bound to uphold the professional privilege of the
attorney-client privilege.
C. Key Questions
1. What is the relationship?
2. What is protected?
3. Who owns/waiver?
4. Exceptions?
II. Attorney-Client Privilege
A. Attorney-Client Key Questions
1. Relationship Attorney-Client
2. Protected -- Confidential communications concerning legal
advice
3. Owns -- Client/waived by disclosure/fail to object
4. Exceptions -- crime, fraud/joint defense
B. Who is an attorney?
1. Must a person be admitted to the bar?
2. In a corporate setting, sometimes it is difficult to tell whether a person with a JD is acting as an
attorney or as a business person
C. Who is a client?
1. Need not have a retainer agreement
2. In a corporate setting, sometimes it is difficult to determine who the client is
D. Confidential Communication Legal Advice
1. Communication must be made with an expectation of confidentiality
2. Statements made in public places that are overheard may not qualify
a. Statements made with third persons present, not necessary to the attorney-client relationship, are
not privileged.
3. Only communications are privileged
a. Handing an attorney pre-existing documents for the attorney to store does not protect the
documents.
b. An attorneys observations are not protected by the privilege.
4. The communication must related to legal advice as opposed to business advice or family matters.
48
Downloaded From OutlineDepot.com

a. Statements made to lawyers are presumed to be made for legal advice, but the context and content
may show otherwise
b. Again the issue comes up frequently in context of corporate counsel when general counsel is
providing employment, business, investment, as well as legal advice
i. Examples: time and place of trial are not privileged, telling client to get rid of gun was
not legal advice, communications sent out that include the PR dept. are usually not legal advice
E. Corporate Setting
1. In corporate setting, questions arise as to when is a JD acting as a lawyer and who are the clients.
2. JDs do lots of tasks in a corporation ranging from accounting, employment advice, budgeting, investment
advice, purchasing and legal advice.
a. Do communications from all employees qualify for privilege? Who is a client?
3. In 1962, a Federal Dist. Ct. in Illinois held that the attorney-client privilege was a personal privilege that
could not be asserted by a corporate entity
a. The decision became overruled but the scope of any privilege became the source of disagreement.
4. Corporate Privilege 2 tests developed
a. Control Group Test
i. privileges tend to keep away valuable evidence
ii. construed narrowly because they obstruct the truth
iii. Attorney-client privilege is to facilitate legal advice
iv. Only those in a position to control decision making in
a corporation need legal advice
b. Subject Matter Test
i. Number of factors to look at:
1) Was communication made to corporate counsel under direction of supervisor?
2) Was ultimate purpose to secure legal advice?
3) Was subject matter within the scope of the employees duties?
4) Was the dissemination limited to those with a need to know to facilitate legal
advice?
5. UpJohn was supposed to reconcile the divergent views and come up with a clear answer. It fails.
a. It establishes the attorney-client privilege as a favored privilege.
b. Rejects the control group test.
c. It does not provide the test to be followed.
d. Court notes the importance of attorney-client privilege:
Its purpose is to encourage full and frank communication between attorneys and their clients and
thereby promote broader public interests in the observance of law and administration of justice.
The privilege recognizes that sound legal advice or advocacy serves public ends and that such
advice or advocacy depends upon the lawyer being fully informed by the client.
F. Rule 501 UpJohn Court instructs the Court to apply privilege rules as governed by the principles of
common law as they may be interpreted by the Courts of the United States in the light of reason and experience.
G. Work Product Doctrine protects the lawyers efforts and thought processes; not limited to
communications
a. Work Product must be materials that are prepared in anticipation of
litigation (for a specific case)
i. No protection for business lawyers planning a merger
ii. Must be more than contemplating litigation prepared or
obtained because of the prospect of litigation
b. Work product may be a qualified privilege
i. If materials are prepared in anticipation of litigation, they need not
be produced absent a showing of substantial need and undue hardship
ii. If the materials reflect thought processes or mental impressions
they are entitled to greater protection may be absolute protection
H. Attorney-Client Privilege Exceptions
49
Downloaded From OutlineDepot.com

1. Crime fraud (Zolin)


2. Joint Defense
3. Will Contests?
Confidentiality and Confidential Communications
I. Attorney-Client Privilege
a. Rule
i. A client has a privilege to prevent the atty from disclosing confidential communication b/w the atty and the
client related to the rendering of legal services
ii. Intended to encourage full disclosure by the client to the atty to facilitate legal counsel
iii. The privilege belongs to the client: only the client can waive the privilege
b. Attorney
i. The communication must have been made to a member of the Bar or to an employee for transmission to the
atty. Communication is protected even if the atty odes not accept the case or the client does not hire the
atty
c. (U.S. v. Woodruff) Attorney-client privilege only extends to legal matters or assistance, it does not cover
non-legal matters
i. The privilege does not cover communications regarding the date and time of the hearing of a defendant.
This is more of a notice function rather than an attorney/client privilege.
ii. Communications that relates to a fact the client told his attorney primarily for the purpose of securing either
an opinion on law, legal services, or assistance in a legal proceeding
iii. The questions posed to Ds counsel do not relate to legal advice, and they will not draw privileged
communication within their scope
iv. Primarily involves the level of loyalty owed to a client and information that could potentially hurt the case
of the client
d. Didnt go over in class! (Upjohn Co. v. U.S.) Disclosures made by corporate employees to corporate
attorneys fall within the attorney-client privilege
i. Documents that reveal the attorneys mental processes in evaluating the information are privileged under
the work-product doctrine
e. (City of San Fran v. Superior Ct.) When a Dr. examines a patient for the sole purpose of aiding an attorney,
atty-client privilege may be invoked
ii. Depends on what the statute says. Must stick to the specific language of the statute. E.g. priest does not
mean pastor. You can also look to case law
iii. How to get around it? Make the doctor an agent of your client. -
iv. Communication from the client to the atty are privileged, this remains true when the communication is via
agent/interpreter
i. When communication to an atty about the clients physical or mental condition requires Dr. assistance to
interpret patients condition, the knowledge obtained by the physician is protected under atty-client
privilege
f. (Clark v. State) Attorney participation in a crime is not protected communication
i. Telephone call between D and attorney. Receptionist listened to the phone call. Privilege goes to the
lawyer, but you cant suppress the information. Exception: Talke of was a crime of fraud.
ii. Public policy does not require protection of a person who takes counsel on how he can safely commit a
crime.
1. This also applies to one who seeks advice on how to destroy evidence
iii. Lawyer technically did not aid the crime, Goldner disagrees w/ decision, at very best the crime was advice
toward destruction of evidence, not murder
g. (U.S. v. Zolin) The party seeking in camera review of evidence under the crime-fraud exception must make
a threshold showing that such review is appropriate
i. Testing proponents claims via in camera review must balance competing interests
1. Too much judicial inquiry would force disclosure of the thing that privilege was meant to protect
2. A complete abandonment of judicial control would lead to intolerable abuses
a. For this reason, in camera review is not an entitlement
50
Downloaded From OutlineDepot.com

ii. Once the showing has been made it is within the courts discretion to grant review
h. (Swidler v. U.S.) blurb, p.600 atty-client privilege survives death and is absolute, no balancing test needed
III. Doctor-Patient Privilege
A. Background
1. Not recognized at CL implemented by statute in NY in 1828
2. Designed to encourage patients to freely disclose everything that will
assist in diagnosis and treatment and to freely seek medical help
3. Not a favored privilege easily waived. But recently with an increased
concern in privacy, it has been revived
B. Relationship
1. Doctor-patient relationship for the purpose of treatment
2. Not applicable if the medical person is consulted for assessment by court
or state, as in drug testing or competency exams or adverse medical exams
in litigation
3. Originally focused on MDs, now expanded in some jurisdictions to
chiropractors, nurses, dentists, licensed social workers
4. Patient need not be conscious but must be alive
5. Privilege survives death
C. Whats Privileged?
1. Original statutes protected information acquired in attending to a patient
in a professional capacity
2. Necessary to enable MD to act in the professional capacity
3. Would include blood tests or observations not limited to
communications.
D. Who Owns/How Waived
1. Owned by patient
2. Cannot be waived by doctor
3. Easily waived by
a. Putting physical condition in controversy (per. injury cases)
b. Failure to object
E. Exceptions
1. Workers compensation
2. Some states limit it in criminal cases
3. Child abuse
4. Some drug abuse
5. Will contests
6. Vulnerable adults
7. Originally no privilege in FRE (Psychotherapy Privilege)
8. Future crimes
I. Physician-Patient
a. Based on the policy of encouraging full disclosure b/w physician and patient
b. Rule A patient, whether or not a party to the action, has a privilege to refuse to disclose and to prevent
physician from disclosing any info acquired by him in confidence
i. Generally the privilege is only assertable in civil actions
ii. The privilege belongs to the patient, but the physician may be compelled to testify if the patient has waived
the privilege
iii. If the patient is absent, the doctor is obligated to assert the privilege for the patient
iv. If the patient is incompetent, the privilege may be asserted by his guardian, or if deceased it may be
asserted by his personal representative
c. Requirements
i. Physicianthe privilege applies if the patient reasonably believes the person is a Doc

51
Downloaded From OutlineDepot.com

ii. Subject matterApplies not only to communications b/w Doc and patient, but also to any info obtained
by the doc in the course of examination regarded as confidential
iii. Professional consultationgenerally, the privilege applies only to info obtained by a physician in the
course of consultation for the purpose of obtaining treatment
1. Thus, when a doc is appointed by the court to make an examination as the basis for testimony is
court, there is NO privilege
d. Exceptions (principal application is in domestic relations cases)
i. Personal injury suit by patient (no privilege)
ii. Competency, guardianship, and commitment proceedings affecting the patient
iii. Deceased patient
1. Privilege does not apply in will contests involving the patients will
iv. Malpractice cases
1. Patient cannot assert privilege to prevent his physician from testifying in any case in which the
patient has asserted a claim against the Doc
v. Illegal purpose
1. No privilege if the services of the physician were sought to assist anyone to plan to commit a
crime or a tort
e. Waiver
i. By contract
1. Usually upheld, but strictly construed
ii. By calling the physician to testify
iii. By patient testifying
1. It is generally held that if the patient testifies as to his consultation with the doctor, he can be
cross-examined, and the Doc may also be compelled
iv. Disclosure to 3rd persons
1. If the communication was made in the presence of unnecessary 3rd persons, the privilege is waived
so that anyone can testify
II. Psychotherapist-Patient
a. Privilege here is greater on the rationale that full disclosure b/w doctor and patient is even more necessary for the
treatment of mental and emotional illness
b. Scope of rule
i. Applies to any litigationcivil or criminaland applies whether or not the patient is a party to the
proceedings
ii. Privilege belongs to the patient
iii. Need not be a physiciancan be a certified psychotherapist (or social worker)
c. Exceptions (broader than physician-patient; i.e. fewer exceptions recognized)
i. Commitment proceedings against the patient
ii. Court-ordered examinations
iii. Mental condition in issue
1. When the patient has placed his mental condition in issue, e.g., by claiming insanity as a defense
d. (Prink v. Rockefeller Center) By bringing a wrongful death action, P waived any privilege arising out of
covering up a suicide attempt
i.
ii. In order to eliminate the unfairness that would result from permitting her to hide behind the privilege, the
better policy is to hold the privilege waived
iii. The material issue of the case was mental statusif the patient was alive he would not have been able to
claim privilege when he put status in issue
e. (Jaffee v. Redmond) Confidential communication b/w psychotherapist and patient in the course of
diagnosis or treatment are protected from compelled disclosure
v. Court recognizes psychotherapist-patient privilege and also extends it to social workers.
vi. The court started with the proposition that privileges are the exception
vii. Mental health of the citizenry was the overriding public value in this case
52
Downloaded From OutlineDepot.com

viii. Scalias Dissent- what if the psychotherapist receives information outside of the scope of the therapy? He
brings up the point that there is no child-mother privilege.
ix. Under 501 the courts may define new privileges by interpreting CL principles in light of reason and
experience
x. This privilege is based on the imperative need for confidence and trust in the relationship. It serves private
interests but also public by facilitating treatment
xi. All states recognize this privilege in one form or another, and to deny a fed privilege would undermine the
effectiveness of the state privileges
1. This applies to social workers as well
f. Menendez v. Superior Court
i. Brothers killed their parents and go to the psychotherapist for help and they threaten to kill the
pyschotherapist
ii. Brotherswere being prosecuted for the murder of their parents
iii. Exception: Dangerous Patient Privilege- in camera review of the cassesttes and notes of psychotherapist
iv. Test: reasonable belief that the patient is dangerous and reasonable belief that disclosure will prevent the
danger.
v. Depends on state whether a third person can reveal what was said.
vi. It must be reasonable belief that the doctor had!
F. Social Worker Privilege Jaffe
1. Applying Rule 501, based on reason and experience, the Court
concluded there is a federal privilege for licensed social worker engaging
in psychotherapy
G. Do Not construe privileges broadly- Must be a licensed psychotherapist!!!
II. Marital Privileges
a. 2 rules:
1) The Marital Testimonial Privilege:The rule prohibiting either spouse from testifying for or against the other, and
2) The Marital Communications Privilege (more common- exists in generally every state): The rule prohibiting
either spouse from revealing confidential communications during marriage
i. Foundation was the marital harmony of the relationship.
b. Marital Testimonial Privilege- Privilege not to testify against spouse
i. At CL, either spouse was disqualified to testify for or against the other
ii. Modern law generally allows either spouse to testify FOR the other in any trial
1. In civil cases, spouses generally are allowed to testify against the other
2. In federal ct., a spouse can testify against the other in criminal cases
iii. Who may assert?
1. Generally, the privilege belongs only to the party-spouse, and therefore the witness-spouse may
testify if party-spouse fails to raise objection
iv. What is excluded?
1. The privilege prevents the witness-spouse from testifying in the present proceeding, but also
excludes any record of the spouses testimony given in any other proceeding (exwhen W was
compelled to testify against H in a civil case, a transcript of that testimony was inadmissible in a
criminal case)
v. Duration
1. The privilege may be asserted only during the marriage
vi. Exceptions (no privilege in the following cases)
1. Crimes against the person or property of the other spouse during the marriage
2. Crimes against the children of either spouse (including failure to support)
3. Certain statutory offenses (importing aliens for prostitution, etc.)
c. The Marital Communications Privilege- Privilege for confidential marital communications (belongs to both spouses)
i. CL rule, still in effect in most states, is that either spouse can refuse to disclose, or can prevent another from
disclosing, confidential communication made b/w spouses during their marriage
ii. Actions in which privilege is assertable
53
Downloaded From OutlineDepot.com

1. Any action, civil or criminal, and not limited to actions where the other spouse is a party (exif
W is called as a witness in litigation b/w P and D, H may appear and assert privilege to prevent W
from revealing any confidences, even if W wants to testify
iii. Who may assert?
1. Privilege belongs to both spouses, either may assert it to avoid giving testimony or to preclude
testimony by the other
iv. What is privileged?
1. Applies only to confidential information
2. Confidential Nature
a. Communication must have been made outside the presence of 3rd party
b. The presumption is confidentiality, the burden is on the party claiming non-
confidentiality to prove it wasnt
3. Observations by spouse
a. No privilege applies as to either spouses observations as to the physical or mental
condition, actions or conduct, of the other spouse b/c there is no communication
involved
d. (Trammel v. U.S.) Spousal immunity has been abandoned, and disclosure of actions and 3rd party communication
is not privileged if she wishes to testify, privilege is witness (hers) alone
i. Pre-Trammel- was the D that the asserted the privilege
ii. Post-Trammel- now it has changed over time that it is the witness spouse that asserts the privilege- it is still
this way today.
III. Miscellaneous Privileges- Tbere are no parent-child privileges!!
a. Didnt Go over in class!- Clergy-Penitent
i. Recognized today in a majority of states
ii. Rule A person may refuse to disclose (and prevent clergy from disclosing) any confidential
commn the person made to clergy acting in his role as advisor
1. Only communications are privileged, observations are not
b. Accountant-Client
i. Privilege only recognized in a few states
c. Parent-Child
i. Not recognized
ii. (In re Grand Jury) No parent-child privilege
d. Farber Procedure- News Persons Privilege- Journalist generally are held to the same obligations as an investigator.
Read this case!
e. Reporters shield Law- a privilege to protect sources. If this law exists in the State!- Different if a criminal trial- if
D subpoenas the reporter then the reporter is violating the constitution- the 6th amendment.

CHAPTER 8
WRITINGS

I. Best Evidence Rule- Background


B. There is no requirement that a proponent prove a point by the most
persuasive evidence.
C. Just like you are not required to call the most qualified expert
II. Limited Original Writings Rule
A. Best Evidence Rule- The OWR provides that if the witness if trying to prove the contents of a
writing you must produce the original or fit it within an exception
1. Rule is technical, but when applied can be unforgiving
2. You can have duplicate originals

54
Downloaded From OutlineDepot.com

B. Rule 1001- Original is the writing or recording itself or any counterpart intended to have the same
effect by a person executing or issuing it. An original of a photograph includes the negative or any print
therefrom. If data are stored in a computer or similar device, any printout or other output readable by sight, shown to reflect
the data accurately, is an original.
C. Duplicate is counterpart produced by the same impression as the orginal, or from the same matrix,
or by means of photography, including enlargements and imiatures, or by mechanism or electronic re-
recording, or by chemical reproduction, or by other equivalent techniques which accurately reproduces the
original.
B. Proving the Contents of a Writing
1. Rule is technical and frequently depends on the form of the testimony
2. In Sirico, if witness testifies to opinion based on x-ray, records, etc., then
no OWR problem but if witness says I looked at x-ray and it
showed then expert is testifying as to the content of the writing
3. To prove payment, earnings or value of a partnership, you need not
produce the books and records.
a. But if the witness testifies that the books and records show
payment, earnings or value, then the witness is testifying as to
what they show proving contents OWR applied
4. To prove what a person said, anyone with personal knowledge can testify
without invoking OWR
a. To prove what the transcript says invokes the OWR (Meyers)
5. To prove what a film depicts implicates the OWR (Enskat)
C. OWR Exceptions
1. Rule 1003 states that duplicates are readily admissible unless the adverse
party objects and raises.
a. Genuine question about authenticity; or
b. Points to circumstances where it would be unfair to admit the
duplicate.
c. Burden is placed on objector to come up with a reason not to
accept the duplicate
2. Rule 1004- Original is not required, and other evidence of the contents of a writing,
recording, or photograph is admissible if:
3. Rule 1004- Original lost or destroyed absent bad faith
3. Rule 1004- Original not obtainable
4. Rule 1004- Original in possession of opponent who was put on notice
5. Rule 1004- Collateral matters (police badge number, writing on Greyhound bus, etc.)
6. Public records Rule 1005
7. Summaries Rule 1006
Writings
I. Best Evidence Rule
a. To prove the contents of a private writing, the original writing must itself be produced, unless it is shown to be
unavailable
i. Slight difference in written words may make a vast difference in meaning, also, production of originals
prevents fraud and mistake, which can occur if copies are used
b. (Sirico v. Cotto) Testimony concerning the contents of x-rays cannot be introduced in evidence w/o
introducing the x-rays
i. The best evidence rule requires a party who seeks to prove the contents of a document to offer in evidence
the original copy of that document unless the absence of the document can be adequately explained
ii. A document within the meaning of the rule includes a letter, contract, receipt, a book, a blueprint, or an x-
ray
c. Waiver by failure to object
i. The best evidence rule is waived by failure to make a timely and specific objection
55
Downloaded From OutlineDepot.com

d. Limitations on the rule


i. Not applicable to official records the rule only applies to private writings, thus, properly authenticated
copies of any official document may be used in lieu of original
ii. Not applicable if contents of writing are collateral also, the rule applies only when the secondary
evidence is offered to prove contents of an original writing
e. (Herzig v. Swift) When contents of a writing itself are not being sought proved, but there is a writing that
contains info to be proved, BER does not require the intro of the writing
i. BER amounts to little more than the requirement that the contents of a writing must be proved by
introduction of the writing itself
ii. Here, the issue was the earnings of the partnership and not the contents of the books, thus the BER would
not apply
f. (Meyers v. U.S.) BER does not apply to oral testimony
i. BER applies to writings.
ii. Prosecution was not trying to establish the contents of a writing but what D said, therefore the BER does
not apply
g. (People v. Enskat) Motion pictures come within the best evidence rule
i. A slide picture is subject to the best evidence rule. A motion picture is a series of these slides and would
also be subject to BER
III. Authentication
A. Background
1. All offers of evidence, be they writings or chattels, must be authenticated
as a conditioned precedent to admissibility
2. Unless self-authenticating, the proponent must offer some evidence from
which the jury could believe the evidence is what the prop. claims it is
3. Rule 901(b) provides non-binding illustrations
B. Authenticating Chattels
1. Can be authenticated by testimony from someone with personal
knowledge Rule 901(b)1 or 4
2. If chattel is fungible (drugs) it could be authenticated by evidence of a
chain of custody Rule 901(b)9
C. Authenticating Writings
1. Can be authenticated by.
a. Personal knowledge
b. Handwriting identification Rule 901(b)2 or 3
c. Circumstantial evidence reply letter doctrine Rule 901(b)9
2. In Dockins, the court seems a bit technical in ruling there is no evidence to
authenticate the fingerprint card came from the Denver PD.
D. Authenticating Conversations
1. Personal knowledge I spoke with Bob, etc.
2. If over the telephone, you must have personal knowledge of the voice of
the person who answered or prove the identity through circumstantial evidence Rules
901(b)1, 4, 5, or 6.
a. Made the call to the assigned number + self identification and
other circumstances
b. Received the call voice identification or strong circumstantial
evidence (reply doctrine)
3. Relevant, not prejudicial, no hearsay problem, best evidence, authentication issues
4. Digital Evidence- physical chain of custody and Logical (data) chain of custody (you
must prove that your copy of the data was taken from the computer in issue)
E. Self-Authentication
1. Some documents need no proof of authentication

56
Downloaded From OutlineDepot.com

2. The documents may still be subject to other objections, hearsay, privilege,


relevancy, etc.
a. Domestic documents under seal
b. Domestic documents, not under seal
b. Foreign public documents
c. Certified copies of Public Records and Official publications of
public authority
d. Newspapers and periodicals
e. Trade inscriptions Green Giant Brand Great Big Sweet Peas
f. Certified Copies of business records
g. Statutory authentication provisions
I. Authentication
a. Rule Before any writing may be received, it must be authenticated, i.e., the proponent must offer a foundation
of evidence sufficient to support a finding that the document is genuine and what it purports to be. FRE 901
i. Not required if genuineness is admitted (in the pleadings or by other evidence, or if the adverse party
fails to raise timely objection to lack of foundation)
ii. Prima facie showing sufficient. Authentication requires only enough evidence to establish a prima facie
showing that the document is what it purports to be
b. (U.S. v. Dockins) A court may not admit a purportedly official document w/o testimony from the custodian
of the document or any circumstantial evidence of its source
i. Fingerprint card from Denver police department was not authenticated. Not admissible because no other
evidence to support its authentication.
ii. The documents were not self-authenticating, so the exhibit could be admitted only under 901. This
requires sufficient evidence to support a finding that the matter in question is what the proponent claims
c. (First State Bank of Denton v. MD Casualty) A phone call is authenticated if the person answering the call
does not identify himself but does identify the residence as the expected residence
i. 901(b)(6) provides for authentication of a phone call when a call is made to the number assigned the
phone company
ii. All that is necessary in authenticating a phone call is that the proponent offer sufficient
authentication to make a prima facie case that would allow the issue of identity to be decided by the
jury.
iii. It is sufficient if the circumstances create a prima facie case that would allow the issue of identity to be
decided by a jury

Anything Under this will not be covered on the Exam!!!

57
Downloaded From OutlineDepot.com

CHAPTER TWELVE
I. Expert Testimony
A. Rule 701 - Opinion Testimony of Lay Witnesses
1. Limited to testimony in the form of opinions or inferences based on
personal knowledge, which are.
a. Rationally based on the perception of the witness
b. Helpful to a clear understanding of the witness testimony
c. Not based on scientific, technical or other specialized knowledge
2. Exampled of Common Law Opinions
a. Speed e. Value of Property
b. Emotions f. Distance
c. Size g. Insanity
d. Intoxication
B. Expert Testimony
1. FRE streamlined the process for admitting expert testimony
a. Requires less formality
b. Trend toward admissibility
c. Reduce emphasis on hypothetical
d. No Rule Against Opinions
C. 3 Basic Areas of Expert Testimony
1. Subject Matter Whether scientific, technical or specialized knowledge
would assist the trier of fact?
a. Must be outside the general knowledge of lay jurors
b. Simply telling jurors how the witness would decide the case is not
helpful
c. Telling the jurors the witness believes the victim is telling the
truth, the D was negligent or that the eyewitness was/wasnt
accurate may not be helpful
2. Qualifications Whether this witness has sufficient specialized
knowledge based on training or (maybe and) experience? The rules require qualification
as an expert but there is no definition or standard of an expert under the Federal Rules.
a. Usually established before the opinion
b. May have developed specialized knowledge by training OR
experience
c. The best experts have both, but courts may allow an expert with
only practical experience to testify, but are very careful about
allowing a witness with only training and no practical experience
d. Need not produce the most qualified expert
i. ** Does the expert have sufficient specialized knowledge
to assist the trier of fact? **
e. Experts can give an opinion regarding an item that is not it no

3. Foundation - Whether the testimony is based upon sufficient facts or


data, is the produce to reliable principles and reliability so as to be of
assistance to the trier of fact

58
Downloaded From OutlineDepot.com

a. Based on sufficient facts


b. The product of reliable principles and methods
c. Applied the principles and methods of reliability. Judge act as the
gatekeeper. (Daubert)
I. Expert Opinions
a. FRE 702-706
b. Defined as men of science educated in the art, or persons possessing special or peculiar knowledge acquired from
practical experience
c. Before an expert is allowed to testify, the ct. must determine:
i. Specialized knowledge helpful to a jury
1. Subject matter at issue must go beyond everyday knowledge
ii. Witness specially qualified
iii. Proper basis for opinion
1. Witnesss opinion must be based on matters that experts in the field rely on
iv. Preliminary fact question for a judge
1. Unless the trial ct. is persuaded by the witness qualifications, it is not allowed
d. Expert testimony occasionally mandatory
e. (State v. Odom) An expert may testify regarding an opinion that characterizes the Ds conduct, including
the Ds intent, so long as he doesnt express an opinion as to guilt
i. As long as the expert does not express his opinion of Ds guilt but characterizes Ds conduct based on facts
in light of specialized knowledge, the opinion is not objectionable
ii. An expert may not express a direct opinion that D is guilty of the crime charged, here, Tierney just drew a
conclusion that possession of the drugs was for distribution
iii. The opinion of an expert can be admitted if it relates to a relevant subject that is beyond the understanding
of the average person of ordinary experience, etc.
f. (U.S. v. Scop) Expert witnesses may not offer opinions that embody legal conclusions by using the terms of
the statute defining the crime
i. FRE 704 permits opinion testimony to by the trier of fact, but does not permit experts to offer opinions
embodying legal conclusions
ii. Testimony was also improper b/c he testified that his opinions were based on a positive assessment of
Sarcinellis trustworthiness
1. Witness credibility is to be determined by the jury
iii. The key question was whether Whitten had relied on the credibility of Sarcinelli. He answered in the
affirmative, thus disqualifying him
g. (Ingram v. McCuiston) Hypo questions to an expert witness cannot include facts that are not in evidence, be
based in part on another experts opinion, or be argumentative
i. To be competent, a hypo may include only facts that are already in evidence or those which the jury might
logically infer
ii. The inclusion of such opinions violates the rule that the opinion of an expert may not be predicated on the
opinions or conclusions of other witnesses, whether expert or lay
iii. CL says that the facts need to be admissible to be allowed in a hypo, BUT FRE 703 now allows facts that
are inadmissible to allow for an opinion or inference to be admitted
h. (U.S. v. Brown) Expert testimony that is based on his own experience in conjunction w/ information and
expertise from non-testifying parties is admissible if it is of a type that is reasonably relied upon by experts in
the field
i. FRE 703 allows experts to rely on otherwise inadmissible data where it has been shown that it is a type
reasonably relied on by experts in the field
ii. The expert is considered to be competent to judge for himself the reliability of the sources on which he
bases his opinions
i. (U.S. v. Tran Trong Cuong) Experts cannot bolster their opinion with hearsay
D. Experts Knowledge
1. Expert can rely on
59
Downloaded From OutlineDepot.com

a. Personal knowledge
b. Facts learned through testimony
c. Hypothetical facts; OR
d. Facts learned prior to trial
2. Rule 703- An expert can rely on facts not admissible at trial if of a type reasonably
relied upon by other experts
a. The inadmissible foundation should not be disclosed to the jury by
the proponent unless the probative value in evaluating the experts
opinion substantially outweighs the prejudicial effect.
3. Rule 705 proponent may provide the opinion without first laying the
elaborate foundation setting forth the facts or data
a. The rule contemplates a situation where the adverse party has
access to the facts or data supporting the opinion and can
effectively engage in cross examination
b. The rule de-emphasizes reliance on the hypothetical question
4. If the issue requires specialized knowledge, the proponent may have to
produce an expert testimony to establish a prima facie case.
a. Example: prove standard of care for brain surgery, the cause of
death, whether a car was negligently designed, etc.
b. If the issue can be resolved by resort to general knowledge, expert
testimony would not be required
i. Example: Doctor leaving an instrument inside a body or
operation on the wrong leg
E. Rule 704 Ultimate Issue
1. Witness can testify about ultimate issues, if helpful
2. Opinion framed in undefined legal terms is usually not helpful
3. Unexplained legal conclusions do not assist the trier of fact.
a. While witness may testify that D was driving fast, they should not testify that Ds negligent
driving caused the accident.
4. In criminal case, Rule 704(b) puts limits on testimony about Ds mens rea
a. Witness may testify that D had a disease or defect and if it could affect his ability to appreciate
nature and consequences of wrongfulness
b. Witness may not testify that D was legally insane/sane or that D did/did not appreciate the nature
and consequences
F. Scientific Evidence
1. Frye Test (1923) test of general acceptance
a. the thing from which the deduction is made must be sufficiently established to have gained
general acceptance in the particular field in which it belongs
b. Left many questions which group of scientists? What is general acceptance? Which evidence
must qualify? How do you accommodate new advances in science?
2. Daubert v. Merrell Dow (1993)- Changed the Test to test of general acceptance to an
evidence test (is it relevant and reliable- the judge makes the decision)
a. Can Bendectin cause birth defects?
i. 30 epidemiological studies do not establish a connection
ii. Plaintiffs experts will testify yes based on:
1. Test tube studies
2. Pharmacological studies
3. Animal studies
4. Unpublished reanalysis of published studies
b. Rejects the Frye Test for Federal Courts
c. Test: Trial judge must serve as gatekeeper to screen unreliable evidence

60
Downloaded From OutlineDepot.com

i. Opinion must be based on science grounding in methods and procedures of science


(testable)
ii. Factor analysis including whether:
1. Tested
2. Subject to Peer Review (publication)
3. Error rates, procedures
4. General Acceptance
d. Daubert Progeny GE v. Joiner (1997)
i. Gatekeeper role extends to
1. Whether the experts opinion reliably flows from the foundation
2. Appellate courts should defer to trial judges

Scientific & Demonstration Evidence


I. (Daubert v. Merrell Dow Pharmaceuticals) As long as the subject matter of an experts testimony is scientific,
technical, or other specialized knowledge, it should be admitted if relevant. There is no requirement for general
acceptance
a. 702 governs expert testimony as long as the specialized knowledge will assist the trier of fact to
understand the evidence, a qualified expert can testify about it
b. There is no requirement for general acceptance as an absolute prerequisite to admissibility of expert testimony
c. Whether a theory or technique has been tested or subjected to peer review can used as a factor in determining the
weight of evidence, but not admissibility
3. U.S. v. Starzecpyzel
a. Forensic Document Examiner testifies that signature is forged -
Court holds that handwriting analysis is not a science
i. Daubert does not apply experiential expertise is helpful
4. Kumho Tire extends trial judges gatekeeping role to all expert testimony
a. Distinction between scientific and technical or specialized
knowledge is not clear
b. Expert, whether scientific or not, must employ in the courtroom
the same level of intellectual rigor that characterizes the practice of an
expert in the relevant field.
c. Reasonable alternative have you tested this? created a
replica/computer model?
5. Suspect Areas
a. High Vulnerability voice prints, hair, handwriting
b. Moderately High Vulnerability glass, fiber, soil, bitemark
c. Readily Admissible fingerprints

-----------------------------------------------------------------------------------------------------------------

CHAPTER 10
JUDICIAL NOTICE
I. Adjudicative Facts
A. Rule 201 governs judicial notice on adjudicative facts only
B. What types of adjudicative facts?
1. Must be one not subject to reasonable dispute in that it is either....
a. Generally known in the jurisdiction, or
i. Example:: Mission St. is downtown San Francisco
IDS Tower is in Hennepin County
61
Downloaded From OutlineDepot.com

Water travels downhill


b. Capable of ready determination
i. Example:: November 28, 1946 was a Thursday
City council ordinance
Religious beliefs of a particular sect
Stock value on a particular day
C. Effect of Judicial Notice
1. Civil case conclusive
2. Criminal case permissible inference
D. When can or should a court take judicial notice?
1. Sua sponte (no one asking)
2. If asked and provided with materials mandatory
3. At any stage of the proceedings even on appeal
II. Stipulations can resolve disputed issue and Judical Notice cannot resolve an issue that is disputed
III. Presumptions
A. Presumptions Burden of Proof
1. Burden of proof is an ambiguous term that includes:
a. Burden of persuasion, or
b. Burden of production
B. Burden of Persuasion (doesnt shift)
1. Relates to the jurys function to decide disputed facts
2. Describes the extent to which the jurors must be convinced in order to rule for the party
3. The burden of persuasion is fixed and does not shift during trial
4. In criminal cases, the state must prove each element of the crime beyond a reasonable
doubt
5. In civil cases, usually the plaintiff must prove the elements by a preponderance of the
evidence
6. Certain disfavored actions (fraud, oral contracts) might require proof by clear and
convincing evidence
7. In both criminal and civil cases, the D may have a burden of persuasion on an affirmative
defense
8. Effectuated when the trial judge instructs the jury
C. Burden of Production (shifts)
1. Relates to judges function as to whether to direct a verdict or send the issue to the jury
2. It is expressed in terms of whether the proponent (one with burden of persuasion) has
produced enough evidence so that a reasonable juror could find for the proponent
3. Burden of production can shift throughout the trial
4. In criminal cases, the accused may have a burden of raising evidence on an affirmative
defense, but never the burden of production on the elements of the crime.
5. Usually starts with the plaintiff who must produce sufficient evidence to present a jury
question to allow the issue to be resolved by the jury
6. The plaintiff might produce overwhelming evidence on the issue that shifts the burden of
production to the adverse party
D. Presumption
1. Assumption of fact required by law upon proof of basic facts
a. If basic facts are proven, then the assumed fact is presumed
i. If there is proof that a letter us stamped, addressed properly and mailed, it is presumed to
have been delivered
2. Effect of Presumption Bursting Bubble
a. FRCP 301 states that presumptions only affect the burden of production
b. A presumption satisfies the burden of production and shifts it to the adverse party.
c. If adverse party doesnt rebut the presumption = directed verdict
62
Downloaded From OutlineDepot.com

d. If adverse party rebuts presumption, it disappears bubble is burst

63

S-ar putea să vă placă și